Séries numériques

Généralités, calculs

($\star$) Correction

Étudier la convergence de série ci-après, puis calculer leur somme si elle converge.

  1. $\dsp\dsum_{n\geq 0}2^{-n}3^{n-1}$
  2. $\dsp\sum_{n\geq 1}\ln\left(\frac{n}{n+1}\right)$
  3. $\dsp\sum_{n\geq 0}\dfrac{1}{4n^2-1}$
  4. $\dsp\sum_{n\geq 0}\dfrac{-1}{9n^2+3n-2}$
  5. $\dsp\sum_{n\geq 0}\dfrac{5}{(5n+2)(5n+7)}$
  6. $\dsp\sum_{n\geq 0}\dfrac{1}{\sqrt{n}+\sqrt{n+1}}$
  7. $\dsp\dsum_{n\geq 0}\dfrac{3}{4^{n-1}}$
  8. $\dsp\dsum_{n\geq 0}\dfrac{3}{(-4)^{n-1}}$
  9. $\dsp\dsum_{n\geq 0}\left(\frac{-1}{\sqrt{5}}\right)^{n}$
  10. $\dsp\sum_{n\geq 0}\dfrac{-2}{(2n+5)(2n+3)}$
  11. $\dsp\dsum_{n\geq 0}(-5)^{n-1}4^n$
  12. $\dsp\dsum_{n\geq 0}37(100)^{-n}$
Correction

Attention à l'ordre!

  1. Soit $N>0$, on a: $$\begin{array}{lcl} \dsp\sum_{ 0\leq n\leq N}\dfrac{-2}{(2n+5)(2n+3)}&=&\dsp\sum_{ 0\leq n\leq N}\left(\dfrac{1}{(2n+5)}-\dfrac{1}{2n+3}\right)\\ &=&\dsp\sum_{ 0\leq n\leq N}\dfrac{1}{2n+5}-\sum_{ 0\leq n\leq N}\dfrac{1}{2n+3}\\ &=&\dfrac{1}{2N+5}-\dfrac{1}{3} \end{array}$$ donc $$\boxed{\dsum_{ 0\leq n\leq N}\dfrac{-2}{(2n+5)(2n+3)}\tendvers{N}{\infty}\dfrac{-1}{3}}.$$

  2. On vérifie que $\dfrac{5}{(5n+2)(5n+7)}=\dfrac{1}{5n+2}-\dfrac{1}{5n+7}$, puis pour $N>0$, on a $$\sum_{n=0}^N\dfrac{5}{(5n+2)(5n+7)}=\sum_{n=0}^n\left(\dfrac{1}{5n+2}-\dfrac{1}{5n+7}\right)=\dfrac{1}{2}-\dfrac{1}{5N+7}.$$ Donc $\dsum_{n=0}^N\dfrac{5}{(5n+2)(5n+7)}\tendvers{N}{\infty}\dfrac{1}{2}$, ce qui donne $$\boxed{\dsum_{n\geq0}\dfrac{5}{(5n+2)(5n+7)}=\dfrac{1}{2}}.$$

  3. On vérifie que $\dfrac{1}{(4n^2-1)}=\dfrac{1}{2}\left(\dfrac{1}{2n-1}-\dfrac{1}{2n+1}\right)$, puis pour $N>0$, on a $$\begin{array}{lcl} \dsp\sum_{n=0}^N\dfrac{1}{(2n+1)(2n-1)}&=&\dsp \sum_{n=0}^n\left(\dfrac{1/2}{2n-1}-\dfrac{1/2}{2n+1}\right)\\ &&=\dfrac{-1}{2}-\dfrac{1}{4N+2}\tendvers{N}{\infty}\dfrac{-1}{2}. \end{array}$$ D'où $$\boxed{ \dsum_{n\geq 0}\dfrac{1}{(2n+1)(2n-1)}=\dfrac{-1}{2}}.$$

  4. On a $9n^2+3n-2=9(n+2/3)(n-1/3)$, on écrit $$\dfrac{-1}{9n^2+3n-2}=\dfrac{a}{3n+2}+\dfrac{b}{3n-1}=\dfrac{1/3}{3n+2}-\dfrac{1/3}{3n-1}$$ puis pour $N>0$, $$\begin{array}{lcl} \dsum_{n=0}^N\dfrac{-1}{9n^2+3n-2}&=&\dsum_{n=0}^N\dfrac{1/3}{3n+2}-\dsum_{n=0}^N\dfrac{1/3}{3n-1}\\ &=&\dfrac{1}{3}+\dfrac{1}{9N+6}\tendvers{N}{\infty}\,\dfrac{1}{3}. \end{array}$$ Conclusion, la série est convergente et $$\boxed{ \dsum_{n\geq 0}\dfrac{-1}{9n^2+3n-2}=\dfrac{1}{3}}.$$

  5. Soit $N>1$, on a $$\sum_{1\leq n\leq N}\ln\left(\frac{n}{n+1}\right)=\sum_{n= 1}^N\left(\ln(n)-\ln(n+1)\right)=-\ln(N+1)\tendvers{N}{\infty}-\infty.$$ Donc $$\boxed{\dsum_{ n}\ln\left(\frac{n}{n+1}\right) \text{ est divergente.}}$$

  6. $\dsp\sum_{n\geq 0}\dfrac{1}{\sqrt{n}+\sqrt{n+1}}$ est divergente.
    En effet, pour $N>0$, on a $$\begin{array}{lcl} \dsum_{n=0}^N\dfrac{1}{\sqrt{n}+\sqrt{n+1}}&=&\dsum_{n= 0}^N\dfrac{\sqrt{n}-\sqrt{n+1}}{n-(n+1)}\\ &&\\ &=&\dsum_{n= 0}^N(\sqrt{n+1}-\sqrt{n})=\sqrt{N+1}\tendvers{N}{\infty}\infty. \end{array}$$ Donc $$\boxed{\dsum_{ n}\dfrac{1}{\sqrt{n}+\sqrt{n+1}} \text{ est divergente.}}$$

  7. $\dsp\dsum_{n\geq 0}\dfrac{3}{4^{n-1}}$ est convergente (série géométrique), plus précisément, on a: $$\boxed{\dsp\dsum_{n\geq 0}\dfrac{3}{4^{n-1}}=12\dsp\dsum_{n\geq 0}\dfrac{1}{4^n}=\dfrac{12}{1-\frac{1}{4}}=16}.$$

  8. $\dsp\dsum_{n\geq 0}\dfrac{3}{(-4)^{n-1}}=-12\dsum_{n\geq 0}\dfrac{1}{(-4)^{n}}$ donc $\boxed{\text{la série converge et sa somme est }\dfrac{-48}{5}}$.

  9. $\dsp\dsum_{n\geq 0}\left(\frac{-1}{\sqrt{5}}\right)^{n}$
    Il s'agit d'une série géométrique de raison $q=\dfrac{-1}{\sqrt{5}}$, donc elle converge puisque $\abs{q}< 1$ et on a: $$\boxed{\dsum_{n\geq 0}\left(\frac{-1}{\sqrt{5}}\right)^{n}=\dfrac{1}{1+\frac{1}{\sqrt{5}}}=\dfrac{5}{5+\sqrt{5}}.}$$

  10. On a $\dsp\dsum_{n\geq 0}2^{-n}3^{n-1}=(1/3)\dsum_{n\geq 0}\left(\frac{3}{2}\right)^n$ donc la série est divergente.

  11. $\dsp\dsum_{n\geq 0}(-5)^{n-1}4^n=\dfrac{-1}{5}\dsum_{n\geq 0}(-20)^n$ donc la série est divergente.

  12. $\dsp\dsum_{n\geq 0}37(100)^{-n}$.
    Il s'agit d'une série géométrique de raison $q=\dfrac{1}{100}$, donc elle converge puisque $\abs{q}< 1$ et on a: $$\boxed{\dsum_{n\geq 0}37\left(\frac{1}{100}\right)^{n}=\dfrac{37}{1-\frac{1}{100}}=\dfrac{3700}{99}.}$$

($\star$) Correction

Soient $\dsum a_n$ et $\dsum b_n$ deux séries convergentes (resp. divergentes), est-ce que $\dsum a_nb_n$ est convergente (resp. divergente)? Justifier.

Correction

Essayer avec $a_n=b_n=\dfrac{(-1)^n}{\sqrt{n}}$ (resp. $a_n=b_n=\dfrac{1}{n}$).

($\star\star$) Correction

Soit $(a_n)_{n\in \N^*}$ une suite de réels définie par: $a_1>2,\quad \forall n\geq 1,\,\,a_{n+1}=a_n^2-2.$

  1. Montrer que $a_n\tendversN\,\infty$.
  2. Montrer que la suite $\left(\dfrac{a_n}{a_1a_2\cdots a_{n-1}}\right)_{n\geq 2}$ converge et calculer sa limite.
  3. Montrer que $\dsum \dfrac{1}{a_1a_2\cdots a_n}$ converge et que sa somme est égale à $\dfrac{a_1-\sqrt{a_1^2-4}}{2}.$

Correction

  1. Il suffit de montrer que si $a_1=2+\eta$ alors $a_n\geq 2+ 2^{n-1}\eta\tendversN \infty$.
    On peut montrer cette relation par récurrence sur $n\in \N^*$.
    La relation est vrai pour $n=1$ car $a_1 =2+\eta \geq 2+2^0\eta$.
    Soit $n\geq 1$, supposon que $a_n\geq 2+2^{n-1}\eta$, alors $$a_{n+1}=a_n^2-2\geq (2+2^{n-1}\eta )^2-2 = 4 +2^{n+1}\eta +4^{n-1}\eta^2-2=2+2^{n+1}\eta +4^{n-1}\eta^2\geq 2+2^n\eta$$ donc la relation est vraie pour tout $n\in \N^*$.
  2. On écrit, pour $n\geq 2$, $a_n^2-4=(a_{n-1}^2-2)^2-4=a_{n-1}^4-4a_{n-1}^2=a_{n-1}^2(a_{n-1}^2-4)$ ce qui donne: $$\forall n\geq 2,\,\dfrac{a_n^2-4}{a_1^2-4}= \prod_{k=1}^{n-1}\dfrac{a_{k+1}^2-4}{a_k^2-4}=\prod_{k=1}^{n-1}a_k^2$$ donc $$ \dfrac{a_n}{a_1\cdots a_{n-1}}=a_n\sqrt{\dfrac{a_1^2-4}{a_n^2-4}}=\sqrt{\dfrac{a_1^2-4}{1-4/a_n^2}}\tendversN\,\sqrt{a_1^2-4}.$$
  3. Pour $n\geq 1$, on écrit $$\dfrac{1}{a_1\cdots a_n}=\dfrac{1}{2}\dfrac{2}{a_1\cdots a_n}=\dfrac{1}{2}\dfrac{a_n^2-a_{n+1}}{a_1\cdots a_n}=\dfrac{1}{2}\left(\dfrac{a_n}{a_1\cdots a_{n-1}}-\dfrac{a_{n+1}}{a_1\cdots a_n}\right)$$ donc il s'agit d'une série télescopique

($\star$) Correction

$\dsum_na_n$ est une série convergente à termes positifs. On pose, pour $n\in \N$, $u_n=\sqrt{a_{2n}a_n}$.
Étudier la série $\dsum_nu_n$.

Correction

En utilisant la relation $ab\leq \dfrac{a^2+b^2}{2}$, on trouve alors $$\forall n\in \N,~~0\leq \sqrt{a_{2n}a_n}\leq \dfrac{1}{2}\left(a_{2n}+a_n\right).$$ Notons, pour $n\in \N$, $S_n=\dsum_{k=0}^na_{2k}$ et $A_n=\dsum_{k=0}^n a_k$. Puisque $a_k\geq 0$, on a $$ \begin{array}{lcl} \forall n\in \N,\, S_n &=& a_0+a_2+a_4+\cdots +a_{2n-2}+a_{2n}\\ &&\\ &\leq& a_0\textcolor{blue}{+a_1}+a_2\textcolor{blue}{+a_3}+a_4+\cdots +a_{2n-2}\textcolor{blue}{+a_{2n-1}}+a_{2n}\\ &&\\ &=&A_{2n} \end{array} $$ Ce qui donne $$\forall n\in \N,~~0\leq S_n\leq A_{2n}\leq \dsum_{k\geq 0}a_k\Longrightarrow \dsum_{k\geq 0}a_{2k}< \infty.$$ On en déduit alors que la suite $(S_n)_n$ converge (suite croissante + majoré), i.e. $$\dsum a_{2n} \text{converge, ce qui donne }\dsum_{k\geq 0}\dfrac{a_{2n}+a_n}{2} \text{ converge}.$$ On en déduit alors que la série $\boxed{\dsum_{n\geq 0}u_n \text{converge}}$.

($\star\star$) Correction

On considère la suite $(a_n)_{n\in \N}$ (Suite de Fibonacci) définie par: $$a_0=a_1=1,\,\quad\text{ et pour tout }n\geq 0, \,\,a_{n+2}=a_{n+1}+a_n.$$ On note également, pour $n\geq 0$, $S_n=\dsum_{k=0}^na_k^2$.

  1. Montrer les relations suivantes: $$\begin{array}{lcl} \mathbf{a)}\,&& \forall n\in \N,\, a_n=\dfrac{1}{\sqrt{5}}\left( \left(\dfrac{1+\sqrt{5}}{2}\right)^{n+1}-\left(\dfrac{1-\sqrt{5}}{2}\right)^{n+1}\right).\\ \mathbf{b)}\,&&\forall n\in \N^*\,\quad (-1)^{n+1}=a_{n-1}a_{n+1}-a_n^2.\\ \mathbf{c)}\,&&\forall n\in \N,\quad (-1)^{n+1}=a_{n+1}a_{n+2}-a_na_{n+3}.\\ \mathbf{d)}\,&&\forall n\in \N,\,\,S_n=a_na_{n+1}. \end{array}$$
  2. Calculer la somme des séries suivantes, $$\mathbf{a)} \quad \dsum_{n\geq 0}\dfrac{(-1)^n}{S_n},\quad\mathbf{b)}\quad\dsum_{n\geq 0}\dfrac{(-1)^n}{a_na_{n+2}},\quad \mathbf{c)}\quad \dsum_{n\geq 1}\arctan \left(\frac{1}{a_{2n}}\right).$$ indication $\arctan(a)+\arctan(b)=\arctan(\cdots)$

Correction

  1. Par récurrence sur $n\in \N$...
  2. $\mathbf{a)} $ Pour $n\in \N^*$, on a $$\dfrac{(-1)^n}{S_n}=\dfrac{(-1)^n}{a_na_{n+1}}=\dfrac{a_n^2-a_{n-1}a_{n+1}}{a_na_{n+1}}=\left(\dfrac{a_n}{a_{n+1}}-\dfrac{a_{n-1}}{a_n}\right).$$ Il s'agit alors d'une série télescopique, pour calculer la somme (on sais déjà qu'elle converge!)
    il faut calculer la limite de $u_n=\dfrac{a_n}{a_{n+1}}$, un simple calcul montre que $\dfrac{a_n}{a_{n+1}}\tendversN \,\dfrac{2}{1+\sqrt{5}}$. On en déduit, $$\boxed{\dsum_{n\geq 0}\dfrac{(-1)^n}{S_n}=\dfrac{2}{1+\sqrt{5}}}.$$ $\mathbf{b)}$ On écrit, $$\begin{array}{lcl} \dfrac{(-1)^n}{a_na_{n+2}}&=&\dfrac{a_na_{n+3}-a_{n+1}a_{n+2}}{a_na_{n+2}}\\ &=&\left(\dfrac{a_{n+3}}{a_{n+2}}-\dfrac{a_{n+1}}{a_{n}}\right)\\ &=&\left(\dfrac{a_{n+3}}{a_{n+2}}-\dfrac{a_{n+2}}{a_{n+1}}\right)+ \left(\dfrac{a_{n+2}}{a_{n+1}}-\dfrac{a_{n+1}}{a_{n}}\right) \end{array}$$ ce qui donne, pour tout $n\geq 1$, $$\dsum_{k=0}^n\dfrac{(-1)^k}{a_ka_{k+2}}=\dfrac{a_{n+3}}{a_{n+2}}-\dfrac{a_2}{a_1}+\dfrac{a_{n+2}}{a_{n+1}}-1=\dfrac{a_{n+3}}{a_{n+2}}+\dfrac{a_{n+2}}{a_{n+1}}-3\tendversN\,\sqrt{5}-2.$$ $\mathbf{c)}$ On écrit, pour $n\geq 0$, $$\begin{array}{lcl} \arctan\left(\dfrac{1}{a_{2n+1}}\right)-\arctan\left(\dfrac{1}{a_{2n+2}}\right)&=& \arctan\left(\dfrac{a_{2n+2}-a_{2n+1}}{1+a_{2n+1}a_{2n+2}}\right)\\ &=&\arctan\left(\dfrac{a_{2n}}{a_{2n}a_{2n+3}}\right)= \arctan\left(\dfrac{1}{a_{2n+3}}\right). \end{array}$$ Donc, pour $n\geq 1$, $\arctan\left(\dfrac{1}{a_{2n}}\right)=\arctan\left(\dfrac{1}{a_{2n-1}}\right)-\arctan\left(\dfrac{1}{a_{2n+1}}\right)$

(CCP PSI 2006) Correction

Nature des séries de terme général $u_n$ et $(-1)^nu_n$ où $$\dsp u_n=n+\dfrac12-\dfrac1{\ln(n^2+n+1)-\ln(n^2+1)}.$$

Correction

On fait un DL $$\begin{array}{cl} \ln(n^2+n+1)-\ln(n^2+1)&=\ln\left(1+\dfrac{1}{n}+\dfrac{1}{n^2}\right)-\ln\left(1+\dfrac{1}{n^2}\right)\\ &\\ & =\dfrac{1}{n}+\dfrac{1}{n^2}-\dfrac{1}{2}\left(\dfrac{1}{n^2}+\dfrac{1}{n^3}\right)+\dfrac{1}{3n^3}-\dfrac{1}{n^2}+ \underset{n\to\infty}{\mathrm{O}}\left(\dfrac{1}{n^4}\right)\\ &\\ &=\dfrac{1}{n}-\dfrac{1}{2n^2}-\dfrac{2}{3n^3}+\underset{n\to\infty}{\mathrm{O}}\left(\dfrac{1}{n^4}\right) \end{array}$$ ce qui donne, $$\begin{array}{lcl} u_n&=&n+\dfrac{1}{2}-\dfrac{n}{1-\dfrac{1}{2n}-\dfrac{2}{3n^2}+\underset{n\to\infty}{\mathrm{O}}\left(\dfrac{1}{n^3}\right)}\\ &=&n+\dfrac{1}{2}-n\left(1+\dfrac{1}{2n}+\dfrac{2}{3n^2}+\dfrac{1}{4n^2} +\underset{n\to\infty}{\mathrm{O}}\left(\dfrac{1}{n^3}\right)\right)\\ &=&\dfrac{-11}{12n}+\underset{n\to\infty}{\mathrm{O}}\left(\dfrac{1}{n^2}\right) \end{array}$$ Donc par comparaison aux séries de Riemann $\dsum u_n$ diverge.

(CCP PC 2006) Correction

Soient $a,b\in\R$. Pour tout $n\in\N^*$, on pose $u_n=\ln(n)+a\ln(n+1)+b\ln(n+2)$.

  1. Montrer que la série $\dsum u_n$ converge si et seulement si $(a,b)=(-2,1)$.
  2. $\mathbf{a)}$ Calculer $\dsum_{n=0}^{\infty} u_n$.
    $\mathbf{b)}$ Soit $\alpha\in\R$. On pose $R_n= \dsum_{k=n+1}^{\infty} u_k$. Nature de la série $\dsum R_n/n^\alpha$ ?

Correction

$\mathbf{1.} $ On fait un DL, $$\begin{array}{lcl} u_n&=&\dsp\ln(n)+a(\ln(n)+\ln(1+1/n))+b(\ln(n)+\ln(1+2/n))\\ &&\\ &=&\dsp (1+a+b)\ln(n)+(a+2b)\frac{1}{n}+(\frac{-1-4b}{2a})\frac{1}{n^2}+\underset{n\to\infty}{\mathrm{O}} (\frac{1}{n^3}) \end{array}$$ Donc, $$\boxed{\sum u_n \text{ converge ssi }(a,b)=(-2,1)}.$$ $\mathbf{2. (a)}$ Dans ce cas, on a $u_n=(\ln(n+2)-\ln(n+1))-(\ln(n+1)-\ln(n))$ il s'agit donc d'une série télescopique, ce qui donne ( puisque $\ln(n+1)-\ln(n)\tendvers{n}{\infty}\,0$) $$\boxed{\dsum u_n=-\ln(2)}.$$ $\mathbf{2. (b)}$ Un calcul simple montre que, pour tout $n\geq 1$, $R_n=-\ln\left(\frac{n+2}{n+1}\right)$. D'où $$ \dfrac{R_n}{n^\alpha}=\dfrac{\ln\left(1+\frac{1}{n}\right)-\ln\left(1+\frac{2}{n}\right)}{n^\alpha}=\dfrac{-1}{n^{\alpha+1}}+\underset{n\to\infty}{\mathrm{O}}\left(\frac{1}{n^{\alpha+2}}\right)$$ Donc, par comparaison aux séries de Riemann $$\boxed{\dsum\dfrac{R_n}{n^\alpha} \text{ converge ssi } \alpha>0}.$$

($\star$--$\star\star\star$) Correction

Étudier les séries de terme général :

  1. $u_n = {\left( 1- \dsp \dfrac{n}{\ln n} \right )}^{-n}$
  2. $u_n=\left(n\sin(1/n)\right)^{n^2}-\ee^{\frac{-1}{6}}$
  3. $u_n = \dsp \dsp \int_0^{+ \infty} \textrm{e}^{-x^n} \ud x$
  4. $u_n=\dsp \dsp \int_0^1 x^n \sin(\pi x) \ud x$
  5. $u_n= \dsp \dfrac{(-1)^n}{\sqrt[n]{n!}}$
  6. $u_n = \ln \left( \dsp\dfrac{\sqrt{n}+(-1)^n}{\sqrt{n+a }}\right)$
  7. $u_n = \dsp \dfrac{1}{\dsp \dsum_{k=1}^n (\ln k)^{\alpha}} \ (n \geqslant 2)$
  8. $x\in \R^*,~~u_n=x^{E(\ln(n))}$
  9. $u_n=\cos \left( \pi\sqrt{n^2+n+1}\right)$

Correction

  1. $u_n = {\left( 1- \dsp \dfrac{n}{\ln n} \right )}^{-n}$.
    On a, pour $n\geq 2$, $\abs{u_n}=\left(\dfrac{n}{\ln n} -1\right )^{-n}$, puisque $n\geq \ln(n)$. Ensuite une étude rapide de la fonction $f(x)=\dfrac{x}{\ln(x)}-1$ (de classe $\CC^1(]1,\infty[)$) montre que, pour tout $x>1$, on a $f(x)\geq \ee-1>\frac{3}{2}$, on en déduit que $\abs{u_n}\leq \left(\frac{2}{3}\right)^n$, puis $\dsum \left(\frac{2}{3}\right)^n$ converge (série géométrique).
    On en déduit que $$\boxed{\dsum_{n\geq 2} u_n \text{ est ACV donc CV}.}$$

    Le critère de Cauchy n'est pas dans le programme de PSI, donc vous n'avez pas le droit d'utiliser ce critère


  2. $u_n=\left(n\sin(1/n)\right)^{n^2}-\ee^{\frac{-1}{6}}$.
    On fait un DL, $$\begin{array}{lcl} u_n&=&\ee^{n^2\ln\left(1-\frac{1}{6n^2}+\mathrm{O}\left(\frac{1}{n^4}\right)\right)}-\ee^{\frac{-1}{6}}\\ &=&-\ee^{\frac{-1}{6}+\mathrm{O}\left(\frac{1}{n^2}\right)}-\ee^{\frac{-1}{6}}\\ &=&\ee^{\frac{-1}{6}}\left(1+\mathrm{O}\left(\frac{1}{n^2}\right)\right)-\ee^{\frac{-1}{6}}=\mathrm{O}\left(\frac{1}{n^2}\right) \end{array}$$ Donc par comparaison avec les séries de Riemann $$\boxed{\dsum u_n \,CV}.$$

  3. $u_n = \dsp \dsp \int_0^{+ \infty} \textrm{e}^{-x^n} \ud x$
    Notons d'abord que l'intégrale définissant $u_n$ est convergente pour tout $n\in \N^*$, en effet, on a, $$\forall n\geq 1,\,\forall x\geq 1,\, x^n\geq x\Longrightarrow 0\leq \ee^{-x^n}\leq \ee^{-x}$$ donc $$ \forall X\geq 1,~~\int_1^X\ee^{-x^n}\ud x\leq \int_1^\infty\ee^{-x}\ud x=\ee^{-1}.$$ Effectuons alors le changement de variable $t=x^n$ (changement de variable valide pour le calcul de $u_n$), on obtient, $$\begin{array}{lcl} u_n&=&\dsp\dfrac{1}{n}\int_0^\infty\ee^{-t}t^{\frac{1-n}{n}}\ud t\\ &\geq &\dsp\dfrac{1}{n}\int_0^1\ee^{-t}t^{\frac{1-n}{n}}\ud t\\ &\geq &\dsp\dfrac{1}{n}\int_0^1\ee^{-t}\ud t=\dfrac{1-\ee^{-1}}{n} \end{array}$$ Comme $\dsum\frac{1-\ee^{-1}}{n}$ diverge, on en déduit que $$\boxed{\dsum u_n\text{ diverge}}.$$

  4. $u_n=\dsp \dsp \int_0^1 x^n \sin(\pi x) \ud x$
    On fait un (deux) intégration par parties $$\begin{array}{lcl} u_n&=&\dsp\dfrac{-\pi}{n+1}\int_0^1x^{n+1}\cos(\pi x)\ud x\\ &=&\dsp\dfrac{\pi}{(n+1)(n+2)}-\dfrac{\pi^2}{(n+1)(n+2)}\int_0^1x^{n+2}\sin(\pi x)\ud x \end{array}$$ On en déduit que $0\leq u_n\leq \dfrac{\pi}{(n+1)(n+2)}$ puisque $\dsp\int_0^1x^{n+2}\sin(\pi x)\ud x\geq 0$.
    Comme $\dsum\dfrac{\pi}{(n+1)(n+2)}$ converge, on en déduit que $$\boxed{\dsum u_n\,\, CV}.$$

  5. $u_n= \dsp \dfrac{(-1)^n}{\sqrt[n]{n!}}$.
    Il s'agit d'une série alterné, on a $u_n\tendvers{n}{\infty}\,0$.
    D'autre part, soit $n\in \N^*$, on a $$\begin{array}{lcl} \dsp\ln(\abs{u_{n+1}})-\ln(\abs{u_n})&=&\dsp-\sum_{k=1}^{n+1}\dfrac{\ln(k)}{n+1}+\sum_{k=1}^{n}\dfrac{\ln(k)}{n}\\ &=&\dsp -\dfrac{\ln(n+1)}{n+1}+\dsum_{k=1}^n\dfrac{\ln(k)}{n(n+1)}\\ &=&\dsp -\dfrac{\ln(n+1)}{n+1}\dsum_{k=1}^n\dfrac{1}{n}+\dsum_{k=1}^n\dfrac{\ln(k)}{n(n+1)} \\ &=&\dsp \dfrac{-1}{n(n+1)}\sum_{k=1}^n\ln\left(\dfrac{n+1}{k}\right)< 0 \end{array}$$ Donc $\ln(\abs {u_{n+1}})< \ln(\abs{u_n})$ c-à-d $\abs{u_{n+1}} < \abs{u_n}$.On en déduit que la suite $(\abs{u_n})$ est décroissante.Donc d'après le CSSA $$\boxed{\dsum u_n\,\, CV}.$$

  6. $u_n = \ln \left( \dsp\dfrac{\sqrt{n}+(-1)^n}{\sqrt{n+a }}\right)$.
    $u_n$ est bien définie pour $n> \abs{a}$. On fait un DL, $$ \begin{array}{ll} u_n&=\dsp\ln\left(\dfrac{1+\frac{(-1)^n}{\sqrt{n}}}{1+\frac{a}{2n}+\mathrm{O}(1/n^2)}\right)\\ &\\ &=\dsp\ln \left( \left( 1+\frac{(-1)^n}{\sqrt{n}}\right) \left(1-\frac{a}{2n}+\mathrm{O}(1/n^2)\right)\right)\\ &\\ &=\dsp\ln\left(1+\frac{(-1)^n}{\sqrt{n}}-\frac{a}{2n}+\mathrm{O}(1/n^2)\right)\\ &=\dsp\frac{(-1)^n}{\sqrt{n}}-\frac{a}{2n}-\frac{1}{2n}+\mathrm{O}(1/n^2) \end{array} $$ On en déduit que $$\boxed{ \dsum u_n \text{ converge ssi } a=-1}.$$

  7. $u_n = \dsp \dfrac{1}{\dsp \dsum_{k=1}^n (\ln k)^{\alpha}} \ (n \geqslant 2)$.
    Il est clair que $\dsum u_n$ diverge si $\alpha\leq 0$. Supposons alors que $\alpha>0$, on a , pour tout $n\geq 2$, $$\int_1^n(\ln(t))^\alpha\ud t \leq \dsum_{k=1}^n (\ln k)^{\alpha}\leq \int_2^{n+1}(\ln(t))^\alpha\ud t $$ donc $$ \dsum_{k=1}^n (\ln k)^{\alpha}\thicksim \int_1^n(\ln(t))^\alpha\ud t\thicksim n\ln(n)^\alpha$$ ce qui donne $u_n\thicksim\frac{1}{n\ln(n)^\alpha}$ , d'où $$\boxed{\dsum u_n \text{ converge ssi }\alpha>1.}$$

  8. $x\in \R^*,~~u_n=x^{E(\ln(n))}$.
    Il faut distinguer plusieurs cas:
    $\mathbf{a) }$ Si $\abs{x}\geq 1$, alors $u_n$ ne tend pas vers $0$ donc $\dsum u_n$ diverge.
    $\mathbf{b) }$ Si $x\in ]0,1[$, alors , $$\forall n\in \N^*,~~x^{\ln(n)}\leq x^{E(\ln(n))}\leq x^{\ln(n)-1}\Longrightarrow n^{\ln(x)}\leq u_n \leq \dfrac{n^{\ln(x)}}{x}$$ ce qui montre que $\dsum u_n$ et $\dsum n^{\ln(x)}$ sont de même nature. Or par comparaison aux séries de Riemann $\dsum n^{\ln(x)}$ converge ssi $\ln(x)>-1$ i.e. $x\in ]0,\frac{1}{\ee}[$.
    $\mathbf{c) }$ Si $x\in ]\frac{-1}{\ee},0[$, alors $\abs{u_n}=\abs{x}^{E(\ln(n))}$ comme $\abs{x}\in ]0, \frac{1}{\ee}[$ alors $\dsum \abs{u_n}$ converge donc $\dsum u_n$ converge.
    $\mathbf{d) }$ Si $x\in ]-1,\frac{-1}{\ee}]$ alors $\dsum u_n$ diverge.
    En effet, pour tout $p\in \N^*$, on note $p_1=E(\ee^p)$ et $p_2=E(\ee^{p+1})-1>p_1$ et si on suppose que $\dsum u_n$ converge, alors en $$ \sum_{n=p_1}^{p_2}u_n=\sum_{n=1}^{p_2}u_n-\sum_{n=1}^{p_1-1}u_n\tendvers{p}{\infty}\,0.$$ Or, $$ \sum_{n=p_1}^{p_2}u_n= \sum_{n=p_1}^{p_2}x^p= \sum_{n=p_1}^{p_2}(-1)^p\abs{x}^p=(-1)^p\abs{x}^p(p_2-p_1+1)$$ ce qui donne $$\abs{\sum_{n=p_1}^{p_2}u_n}\geq \abs{x\ee}^p(\ee-1-\frac{1}{\ee^p})\geq \ee-1-\frac{1}{\ee}\underset{p\rightarrow\infty}{\not\longrightarrow}\,0$$ Contradiction.
    En conclusion $$\boxed{\dsum u_n\text{ converge ssi }x\in ]\frac{-1}{\ee},\frac{1}{\ee}[\setminus\{0\}}.$$

  9. $u_n=\cos \left( \pi\sqrt{n^2+n+1}\right)$.
    On fait un DL, $$\begin{array}{rl} u_n&=\cos\left[\pi n\sqrt{1+\frac{1}{n}+\frac{1}{n^2}}\right]=\cos\left[\pi n\left(1+\frac{1}{2n}+\frac{3}{8n^2}+\mathrm{O}(\frac{1}{n^3})\right)\right]\\ &\\ &=(-1)^n\sin\left[\frac{3}{8n}+\mathrm{O}(\frac{1}{n^2})\right]=\dfrac{3(-1)^n}{8n}+\mathrm{O}(\frac{1}{n^2}) \end{array}$$ Comme la série de terme générale $\dfrac{3(-1)^n}{8n}$ converge (CSSA), et la série de terme générale $\mathrm{O}(\frac{1}{n^2})$ converge, on en déduit que $\dsum u_n$ converge.

($\star$) Correction

Pour $n\in \N^*$, on pose $p(n)$ le nombre de chiffre dans l'écriture de $n$ en base 10.
On pose $u_n=\dfrac{p(n)}{n(n+1)}$. Montrer que la série $\dsum_{n\geq 1}u_n$ converge puis calculer sa somme.

Correction

Un encadrement simple montre que $p(n)\in \left[\frac{\ln(n)}{\ln(10)},\frac{\ln(n)}{\ln(10)}+1\right]$, ainsi $$0\leq u_n\leq \dfrac{\frac{\ln(n)}{\ln(10}+1}{n(n+1)}\Longrightarrow u_n=\underset{n\rightarrow\infty}{\mathrm{o}}\left(\frac{1}{n^{3/2}}\right)\Longrightarrow \boxed{\dsum u_n \text{ converge}}.$$ Remarquons que $\N^*= \dsp \bigcup_{p\geq 0}\inter{10^p,10^{p+1}-1}$, on trouve: $$\begin{array}{lcl} \dsum_{k=1}^{\infty}u_k&=& \dsum_{p=0}^{\infty}\left(\dsum_{k=10^p}^{10^{p+1}-1}\dfrac{p+1}{k(k+1)}\right)\\ &&\\ &=& \dsum_{p=0}^{\infty}(p+1)\left(\dsum_{k=10^p}^{10^{p+1}-1}\dfrac{1}{k}-\frac{1}{k+1}\right)= \dsum_{p\geq 0}\left(\dfrac{p+1}{10^p}-\dfrac{p+1}{10^{p+1}}\right) \end{array}$$ Les séries $\dsum \frac{p}{10^p}$ et $\dsum \frac{p+1}{10^p}$ sont convergentes , donc: $$\dsum_{n\geq 1}u_n=\dsum_{p\geq 0}\dfrac{p+1}{10^p}- \dsum_{p\geq 0}\dfrac{p+1}{10^{p+1}}=\dsum_{p\geq 0} \dfrac{1}{10^p}+\dsum_{p\geq 0}\dfrac{p}{10^p}-\dsum_{p\geq 0}\dfrac{p+1}{10^p} =\dsum_{p\geq 0}\dfrac{1}{10^p} $$ Ce qui donne finalement, $\boxed{ \dsum_{n\geq 1} u_n=\dfrac{10}{9}}$.

(CCP PSI) Correction

Soient $a,b\in \R$. On pose $u_n=\sqrt{n}+a\sqrt{n+1}+b\sqrt{n+2},~~(n\in \N)$. Déterminer $(a,b)\in \R^2$ tel que $\dsp\sum u_n$ converge et calculer sa somme.

Correction

On fait un DL $$\begin{array}{lcl} u_n&=&\sqrt{n}\left(1+a\left(1+\dfrac{1}{2n}+\underset{n\to\infty}{\mathrm{O}}\left(\dfrac{1}{n^2}\right)\right)+b\left(1+\dfrac{1}{n}+\underset{n\to\infty}{\mathrm{O}}\left(\dfrac{1}{n^2}\right)\right) \right)\\ &=&(a+b+1)\sqrt{n}+\dfrac{a+2b}{2\sqrt{n}}+\underset{n\to\infty}{\mathrm{O}}\left(\dfrac{1}{n\sqrt{n}}\right) \end{array}$$ Donc $\dsum u_n$ converge ssi $$\left\{\begin{array}{ccc} a+b&=&-1\\ a+2b&=&0 \end{array} \right.\Longleftrightarrow (a,b)=(-2,1)$$ Dans ce cas, on a $$u_n=\sqrt{n}-2\sqrt{n+1}+\sqrt{n+2}=\left(\sqrt{n}-\sqrt{n+1}\right)-\left(\sqrt{n+1}-\sqrt{n+2}\right)$$ Il s'agit d'une série télescopique, en faisant la somme on obtient $$\boxed{\dsp\sum_{n\geq 0}u_n=-1}$$

($\star$) Correction

Soit $\lambda\in \R$. Montrer que la série $\dsp\sum n^2\dfrac{\lambda^n}{n!}$ est convergente puis calculer sa somme.

Correction

Supposons que $\lambda \neq 0$ (pour $\lambda=0$ le résultat est immédiat). Posons $u_n=\dfrac{n^2\abs{\lambda}^n}{n!}$, on a $$u_n>0\text{ et }\dfrac{u_{n+1}}{u_n}=\dfrac{(n+1)^2\abs{\lambda}^{n+1}}{n^2\abs{\lambda}^n}\dfrac{n!}{(n+1)!}=\dfrac{(n+1)^2}{n^2}\dfrac{\abs{\lambda}}{n+1}\tendvers{n}{\infty}\,0.$$ donc $\dsum u_n$ est convergente ce qui donne $\dsp\sum n^2\dfrac{\lambda^n}{n!}$ est ACV donc CV.
D'autre part, on a $$\forall n\geq 2,~~~\dfrac{n^2\lambda^n}{n!}=\dfrac{n\lambda^n}{(n-1)!}=\dfrac{\lambda^n}{(n-2)!}+\dfrac{\lambda^n}{(n-1)!}$$ ce qui donne $$\dsp\sum_{n=1}^\infty n^2\dfrac{\lambda^n}{n!}=\lambda+\sum_{n=2}^\infty\dfrac{n^2\lambda^n}{n!}=\lambda+\lambda^2\sum_{n\geq 2}\dfrac{\lambda^{n-2}}{(n-2)!}+\lambda \sum_{n\geq 2}\dfrac{\lambda^{n-1}}{(n-1)!}=\lambda^2\dsum_{n\geq 0}\dfrac{\lambda^n}{n!}+\lambda \dsum_{n\geq 0}\dfrac{\lambda^n}{n!}$$ Ce qui donne finalement, $$\boxed{\dsum_{n\geq 0}n^2\dfrac{\lambda^n}{n!}=(\lambda^2+\lambda)\ee^\lambda .} $$

(Mines-Télécom 2017) Correction

Déterminer les valeurs de $a$ et $b$ pour lesquelles la série $\dsum\left(2+(an+b)\ln\dfrac{n+2}{n+4}\right)$ converge.

Correction

On fait un DL, $$\begin{array}{lcl} \dsp\ln\left(\frac{n+2}{n+4}\right)&=&\dsp\ln\left(1+\frac{2}{n}\right)-\ln\left(1+\frac{4}{n}\right) =\frac{2}{n}-\frac{2}{n^2}+\frac{8}{3n^3}-\left(\frac{4}{n}-\frac{8}{n^2}+\frac{64}{3n^3}\right)+\underset{n\to \infty}{\mathrm{o}}\left(\frac{1}{n^3}\right)\\ &&\\ &=&\dsp \frac{-2}{n}+\frac{6}{n^2}+\frac{-56}{3n^3}+\underset{n\to \infty}{\mathrm{o}}\left(\frac{1}{n^3}\right) \end{array}$$ Ce qui donne, $$2+(an+b)\ln\dfrac{n+2}{n+4}=2-2a+\frac{6a-2b}{n}+\frac{18b-56a}{3n^2}+\underset{n\to \infty}{\mathrm{o}}\left(\frac{1}{n^2}\right)$$ Donc $\boxed{ \dsum\left(2+(an+b)\ln\dfrac{n+2}{n+4}\right)\text{ converge ssi }(a,b)=(1,3)}$.

(Vraie/Faux) Correction

Répondre par vraie ou faux aux questions suivantes, justifier vos réponses.

  1. La somme de deux séries divergentes est une série divergente.
  2. Si $\dsum u_n$ converge alors $\dsum \abs{u_n} $ converge.
  3. Si $u_n\backsim v_n$ alors $\dsum u_n$ et $\dsum v_n$ sont de même nature.
  4. Si $u_n$ est de signe constante (pour $ n\geq n_0$) et $\dsum u_n$ converge alors $\dsum u_n^2$ converge.
  5. Si $u_n\tendversN\,0$ et $\dsum v_n$ converge alors $\dsum u_nv_n$ converge.
  6. Si $(a_n)$ positive décroissante telle que $\dsum na_n$ converge alors $\dsum n(a_n-a_{n-1})$ converge.
  7. Si $\dsum u_n$ converge alors $\dsum u_n^3$ converge.

Correction

  1. Faux
  2. Faux
  3. Faux
  4. Vrai
  5. Faux
  6. Vrai
  7. Faux

($\star\star$) Correction

Pour $n\in \N$, on note $w_n=\dsum_{k=1}^nk!$. Montrer que $\dsum u_n $ diverge et $\dsum v_n$ converge. Avec $$u_n=\dfrac{w_n}{(n+1)!},\quad v_n=\dfrac{w_n}{(n+2)!}$$

Correction

Soit $n\in \N,\, n\geq 2$, on a $$\begin{array}{lcl} w_n&=&1+2!+\cdots +n!=n!\left(1+\dfrac{(n-1)!}{n!}+\dfrac{(n-2)!}{n!}+\cdots +\dfrac{2!}{n!}+\dfrac{1}{n!}\right)\\ &=&n!\left(1+\dfrac{1}{n}+\dfrac{1}{n(n-1)}+\cdots +\dfrac{2!}{n!}+\dfrac{1}{n!}\right) \end{array}$$ Pour $j\in \inter{1,n-2}$, on a $ \dfrac{j!}{n!}\leq \dfrac{1}{n(n-1)}$ ce qui donne: $$\forall n\geq 2,\quad n!\leq w_n\leq n!\left(1+\dfrac{1}{n}+\dfrac{n-2}{n(n-1)}\right) \Longrightarrow w_n\backsim n!$$ On en déduit alors: $$u_n\backsim \dfrac{n!}{(n+1)!}=\dfrac{1}{n+1},\quad v_n\backsim \dfrac{n!}{(n+2)!}=\dfrac{1}{(n+2)(n+1)}.$$ Ce qui donne le résultat.

($\star\star$) Correction

Soit $(u_n)$ une suite à terme strictement positifs, décroissante. On suppose que $\dsum u_n$ converge, montrer que $ nu_n\tendversN 0$. Réciproque?

Correction

Notons $R_n=\dsum_{k\geq n+1} u_n$, on a pour tout $n\geq 1$, $$R_{n}-R_{2n}=\dsum_{k=n+1}^{2n}u_k\geq n u_{2n}$$ Donc $nu_{2n}\tendversN 0$, ce qui implique aussi que $2nu_{2n}\tendversN\,0$.
On a aussi pour tout $n\geq 0$, $$0\leq (2n+1)u_{2n+1}\leq (2n+1)u_{2n}=2nu_{2n}+u_{2n}\tendversN\,0$$ On en déduit que les deux suites $(2nu_{2n})$ et $((2n+1)u_{2n+1})$ convergent vers $0$ ce qui implique que $nu_n\tendversN\,0$.
Posons, pour $n\geq 2$, $u_n=\dfrac{1}{n\ln(n)}$, on a $u_n>0 $ et $nu_n\tendversN\,0$ mais $\dsum u_n $ diverge.

($\star$) Correction

Soit $(u_n)_n$ une suite de réels positifs. On pose $v_n=\dfrac{u_n}{1+u_n}$. Montrer que $\dsum u_n$ et $\dsum v_n$ sont de même nature.

Correction

On distingue deux cas:

  1. Si $u_n\tendversN\,0$ alors $u_n\underset{\infty}{\thicksim}v_n$ puisque $\dfrac{v_n}{u_n}=\dfrac{1}{1+u_n}\tendversN\,1$, en utilisant le résultat du théorème (\ref{comp4:thm}), on trouve $\dsum u_n$ et $\dsum v_n$ sont de même nature.
  2. Si $(u_n)$ ne tend pas vers $0$ (donc $\dsum u_n$ diverge), alors $(v_n)$ ne tend pas vers $0$, en effet: $$\forall \varepsilon>0,\,\, \forall n\in \N,\,\exists m\geq n, \,\,u_m>\varepsilon, \text{or } v_m=\dfrac{u_m}{1+u_m}\geq \dfrac{\varepsilon}{1+\varepsilon}>0.$$ (La fonction $x\longmapsto \dfrac{x}{1+x}$ est croissante sur $\R_+$), donc $\dsum v_n$ diverge, i.e. $\dsum u_n$ et $\dsum v_n$ sont de même nature.

($\star$) Correction

Soit $\rho\in ]0,1[,\,\theta\in \R$. Montrer la convergence de $\dsum \rho^n\cos(n\theta)$ (resp. $\dsum \rho^n\sin(n\theta)$) puis calculer leurs sommes.

Correction

On pose $z=\rho\ee^{\ii \theta}\in \C$, on a $\abs{z}=\rho < 1$, d'après le cours, on a $\dsum z^n$ converge et, $$\begin{array}{lcl} \dsum_{n\geq 0} z^n&=&\dfrac{1}{1-z}=\dfrac{1}{(1-\rho\cos(\theta))-\ii \rho\sin(\theta)}\\ &=&\dfrac{(1-\rho\cos(\theta))+\ii \rho\sin(\theta)}{(1-\rho\cos(\theta))^2+ \rho\sin(\theta)^2}=\dfrac{(1-\rho\cos(\theta))+\ii \rho\sin(\theta)}{\rho^2-2\rho\cos(\theta)+1}\\ &=& \dfrac{(1-\rho\cos(\theta))}{\rho^2-2\rho\cos(\theta)+1}+\ii\dfrac{\rho\sin(\theta)}{\rho^2-2\rho\cos(\theta)+1} \end{array}$$ On en déduit, que les séries $\dsum \rho^n\cos(n\theta)$ et $\dsum \rho^n\sin(n\theta)$ convergent et $$\dsum_{n\geq 0} \rho^n\cos(n\theta)=\re (\dsum z^n)=\dfrac{1-\rho\cos(\theta)}{\rho^2-2\rho\cos(\theta)+1},$$ $$\dsum_{n\geq 0} \rho^n\sin(n\theta)=\im (\dsum z^n)=\dfrac{\rho\sin(\theta)}{\rho^2-2\rho\cos(\theta)+1}.$$

Séries alternées

($\star$) Correction

Étudier les séries de terme général :

  1. $\dsp\sum_{n\geq 0}\dfrac{(-1)^{n-1}}{n^2+1}$
  2. $\dsp\sum_{n\geq 1}\dfrac{(-1)^nn}{5^n}$
  3. $\dsp\sum_{n\geq 0}(-1)^n\dfrac{\ee^n-1}{\ee^n+1}$
  4. $\dsp\sum_{n\geq 0}(-1)^n\dfrac{1}{\ln (n+1)}$
  5. $\dsp\sum_{n\geq 1}(-1)^nn\sin(\frac{1}{n})$
  6. $\dsp\sum_{n\geq 1}\dfrac{\sin\left(\frac{2n-1}{2}\pi\right)}{n}$
Correction

  1. $\dsp\sum_{n\geq 0}\dfrac{(-1)^{n-1}}{n^2+1}$ est absolument convergente donc convergente.
  2. $\dsp\sum_{n\geq 1}\dfrac{(-1)^nn}{5^n}$ est absolument convergente donc convergente.
  3. $\dsp\sum_{n\geq 0}(-1)^n\dfrac{\ee^n-1}{\ee^n+1}$ est divergente puisque $(-1)^n\dfrac{\ee^n-1}{\ee^n+1}\underset{n\rightarrow\infty}{\nrightarrow}0$.
  4. $\dsp\sum_{n\geq 0}(-1)^n\dfrac{1}{\ln (n+1)}$ est convergente (CSSA).
  5. $\dsp\sum_{n\geq 1}(-1)^nn\sin(\frac{1}{n})$ est divergente puisque $(-1)^nn\sin(\frac{1}{n})\underset{n\rightarrow\infty}{\nrightarrow}0$.
  6. $\dsp\sum_{n\geq 1}\dfrac{\sin\left(\frac{2n-1}{2}\pi\right)}{n}$ est convergente (CSSA et $\sin\left(\frac{2n-1}{2}\pi\right)=(-1)^n$).
($\star$) Correction

Calculer une valeur approchée à $10^{-3}$ près de la somme des séries suivantes

  1. $\dsp\sum_{n\geq 0}\dfrac{(-1)^{n}}{n!}$
  2. $\dsp\sum_{n\geq 0}\dfrac{(-1)^{n+1}}{(2n)!}$
  3. $\dsp\sum_{n\geq 1}\dfrac{(-1)^n}{n^3}$
  4. $\dsp\sum_{n\geq 1}\dfrac{(-1)^n}{n^5}$
  5. $\dsp\sum_{n\geq 0}\dfrac{(-1)^n}{n^3+1}$
  6. $\dsp\sum_{n\geq 0}\dfrac{(-1)^{n-1}(n+1)}{5^n}$
Correction

Rappelons un résultat de cours:
si une série alternée $\dsum u_n$ CV et vérifie les hypothèses de CSSA alors : $$\forall n\in \N \abs{R_n =\dsum_{k\geq n+1}u_k}\leq \abs{u_{n+1}}.$$ Donc pour répondre à cette question, il faut trouver $n_0\in \N$ tel que $R_{n_0}\leq 10^{-3}$ ensuite calculer $U_{n_0}$.

  1. $\dsp\sum_{n\geq 0}\dfrac{(-1)^{n}}{n!}$. $$\abs{R_n}\leq \abs{u_{n+1}}=\dfrac{1}{(n+1)!}\leq 10^3\Longrightarrow 1000\leq (n+1)!\Longrightarrow n\geq 6.$$ Donc, $$\sum_{n\geq 0}\dfrac{(-1)^{n}}{n!}=1-1+\dfrac{1}{2}-\dfrac{1}{6}+\dfrac{1}{24}-\dfrac{1}{120}+\dfrac{1}{720}+\varepsilon=\dfrac{265}{720}+\varepsilon=\dfrac{53}{144}+\varepsilon.$$ Avec $\abs{\varepsilon}\leq 10^{-3}$. En utilisant $\text{\texttt{Maple}}$, on trouve: $$\begin{array}{lcl} A= \dsum_{n\geq 0}\dfrac{(-1)^{n}}{n!}=\ee^{-1}&=& .367\,879\,441\,171\,442\,321\,596\\ B= \dsum_{n= 0}^6\dfrac{(-1)^{n}}{n!}&=&.368\,055\,555\,555\,555\,555\,556\\ A-B&=&.000\,176\,114\,384\,113\,233\,960 \end{array}$$
  2. $\dsp\sum_{n\geq 0}\dfrac{(-1)^{n+1}}{(2n)!}$.
    Il suffit de prendre $n=3$. $$\dsp\sum_{n\geq 0}\dfrac{(-1)^{n+1}}{(2n)!}=-1+\dfrac{1}{2}-\dfrac{1}{24}+\dfrac{1}{720}+\varepsilon=\dfrac{-389}{720}+\varepsilon$$
  3. $\dsp\sum_{n\geq 1}\dfrac{(-1)^n}{n^3}$
    Il faut choisir $n$ tel que $\dfrac{1}{(n+1)^3}\leq 10^{-3}$, i.e. $n\geq 9$.
  4. $\dsp\sum_{n\geq 1}\dfrac{(-1)^n}{n^5}$
  5. $\dsp\sum_{n\geq 0}\dfrac{(-1)^n}{n^3+1}$ $$\dfrac{1}{(n+1)^3+1}\leq 10^{-3}\Longrightarrow 1000\leq (n+1)^3+1 \Longrightarrow n\geq 9.$$
  6. $\dsp\sum_{n\geq 0}\dfrac{(-1)^{n-1}(n+1)}{5^n}$ $$\dfrac{n+2}{5^{n+1}}\leq 10^{-3}\Longrightarrow 200\leq \dfrac{5^n}{n+2}\Longrightarrow n\geq 5.$$

($\star\star$) Correction

  1. Montrer la convergence de la série de terme générale $u_n=(-1)^n\ln\left(1+\dfrac{1}{n}\right)$. On note $S=\dsum_{n\geq 1} u_n$.
  2. Montrer que, pour tout $n\geq 1$, $S_{2n}$ peut s'écrire $S_{2n}=\ln\left(\dfrac{(2n+1)[(2n)!]^2}{2^{4n}(n!)^4}\right)$ en déduire la limite de $S_{2n}$.
  3. Déterminer $S$.

Correction

  1. D'après le CSSA, $\dsum u_n$ converge, en effet il s'agit d'une série alterné avec $u_n\tendversN\,0$ et $(\abs{u_n})$ est strictement décroissante.
  2. Soit $n\geq 0$, on a : $$\begin{array}{lcl} S_{2n}&=&\dsum_{k=1}^{2n}(-1)^k\ln\left(1+\dfrac{1}{k}\right)\\ &=&\dsum_{k=1}^{n}(-1)^{2k}\ln\left(1+\dfrac{1}{2k}\right)+\dsum_{k=0}^{n-1}(-1)^{2k+1}\ln\left(1+\dfrac{1}{2k+1}\right)\\ &=&\dsum_{k=1}^{n}\ln\left(\dfrac{2k+1}{2k}\right)-\dsum_{k=0}^{n-1}\ln\left(\dfrac{2k+2}{2k+1}\right)\\ &=&\ln\left(\dsp\prod_{k=1}^n\dfrac{2k+1}{2k}\right)+\ln\left(\dsp\prod_{k=0}^{n-1}\dfrac{2k+1}{2k+2}\right)\\ &=&\dsp \ln\left(\prod_{k=1}^n\dfrac{2k+1}{2k}\prod_{k=0}^{n-1}\dfrac{2k+1}{2k+2}\right) \end{array}$$ Or $$\begin{array}{lcl} \dsp \prod_{k=1}^n\dfrac{2k+1}{2k}&=&\dfrac{3}{2}\dfrac{5}{4}\cdots\dfrac{2n-1}{2n-2}\dfrac{2n+1}{2n}\\ &&\\ &=&\textcolor{blue}{\dfrac{2}{2}} \dfrac{3}{2}\textcolor{blue}{\dfrac{4}{4}}\dfrac{5}{4}\cdots\textcolor{blue}{\dfrac{2n-2}{2n-2}}\dfrac{2n-1}{2n-2}\textcolor{blue}{\dfrac{2n}{2n}}\dfrac{2n+1}{2n}\\ &&\\ &=&\dfrac{(2n+1)!}{\textcolor{blue}{4}\times 1^2\times\textcolor{blue}{4}\times 2^2\cdots\textcolor{blue}{4}\times (n-1)^2\times\textcolor{blue}{4}\times n^2 }=\dfrac{(2n+1)!}{4^n (n!)^2} \end{array}$$ Un calcul similaire donne $\dsp \prod_{k=0}^{n-1}\dfrac{2k+1}{2k+2}=\dfrac{(2n)!}{4^n(n!)^2}$, d'où $$S_{2n}=\ln\left(\dfrac{(2n+1)!}{4^n(n!)^2}\dfrac{(2n)!}{4^n(n!)^2}\right)= \ln\left(\dfrac{(2n+1) [(2n)!]^2}{4^{2n}(n!)^4}\right)$$ Pour trouver la limite de $(S_{2n})$, on utilise la formule de Stirling, $$\begin{array}{lcl}\dfrac{(2n+1) [(2n)!]^2}{4^{2n}(n!)^4}&\thicksim&\dfrac{(2n+1) (2n)^{4n}\sqrt{2\pi (2n)}^2}{\ee^{4n}}\dfrac{\ee^{4n}}{4^{2n}(n)^{4n}\sqrt{2\pi\,n}^4}\\ &&\\ &=&\dfrac{(2n+1)2^{4n}n^{4n}(4\pi\,n)}{4^{2n}n^{4n}4\pi^2n^2}=\dfrac{2n+1}{\pi n}\tendversN\,\dfrac{2}{\pi} \end{array}$$ On en déduit, $$\boxed{S_{2n}\tendversN\,\ln\left(\frac{2}{\pi}\right)}.$$
  3. Comme la suite $(S_n)$ converge, alors la limite de $(S_{2n})$ est égale à la limite de la suite $(S_n)$, d'où $\boxed{\textcolor{red}{\dsp S=\ln\left(\frac{2}{\pi}\right)}}$.

($\star\star$) Correction

Étudier la série de terme générale $u_n=\dfrac{(-1)^n}{(n!)^{1/n}}$.

Correction

Notons $v_n=\dfrac{1}{(n!)^{1/n}}$, il est clair que $v_n>0$, d'autre part pour $n\geq 1$, on a: $$\begin{array}{lcl} \ln(v_{n})-\ln(v_{n+1})&=&\dfrac{-1}{n}\dsum_{k=1}^{n}\ln(k)-\dfrac{-1}{n+1}\dsum_{k=1}^{n+1}\ln(k)\\ &=&\left(\dfrac{-1}{n}-\dfrac{-1}{n+1}\right)\dsum_{k=1}^{n}\ln(k)-\dfrac{\ln (n+1)}{n+1}\\ &=&\left(\dfrac{-1}{n(n+1)}\right)\dsum_{k=1}^{n}\ln(k)+\dfrac{\ln (n+1)}{(n+1)n}\dsum_{k=1}^n1\\ &=& \dfrac{1}{n(n+1)}\dsum_{k=1}^n (\ln(n+1)-\ln(k))\geq 0 \end{array}$$ On en déduit alors que la suite $(v_n)$ est décroissante. En utilisant la formule de Stirling, on trouve que $v_n\tendversN\,0$.
On en déduit d'après le critère spéciale des séries alternées, que $\dsum u_n $ converge.

(CCP PSI 2016) Correction

On pose, pour $n\in \N$, $a_n=\dsp\int_0^1t^n\sqrt{1-t^2}\ud t$.

  1. Montrer que la série de terme générale $(-1)^na_n$ converge et calculer sa somme.
  2. Trouver, pour $n\geq 2$, une relation entre $a_n$ et $a_{n-2}$.
  3. Montrer que $n(n+1)(n+2)a_na_{n-1}$ est constant.
  4. En déduire la convergence de $\dsum a_n$.

Correction

  1. Pour $n\geq 0$, on a $$\forall t\in [0,1],\,0\leq t^{n+1}\leq t^n \Longrightarrow 0\leq a_{n+1}\leq a_{n}$$ donc la suite $(a_n)$ est décroissante.
    D'autre part, pour tout $n\in \N$, $$\int_0^nt^n \sqrt{1-t^2}\ud t\leq \int_0^1t^n\ud t=\dfrac{1}{n+1}\tendversN\, 0$$ donc $a_n\tendversN\,0$. En utilisant le critère spéciale des séries alternée, on trouve $\dsum (-1)^na_n$ converge.
    Soit $N\geq 1$, on a $$\forall t\in [0,1],\quad \dsum_{n=0}^N(-t)^n= \dfrac{1-(-t)^{N+1}}{1+t}$$ On en déduit, $$\dsum_{n=0}^N(-1)^na_n=\dsum_{n=0}^N\int_0^1(-t)^n\sqrt{1-t^2}\ud t =\int_0^1\dfrac{\sqrt{1-t^2}}{1+t}\ud t +(-1)^{N+2}\int_0^1\dfrac{t^{N+1}\sqrt{1-t^2}}{1+t}\ud t.$$ D'autre part, $$0\leq \int_0^1\dfrac{t^{N+1}\sqrt{1-t^2}}{1+t}\ud t\leq \int_0^1t^{N+1}\ud t=\dfrac{1}{N+2}\tendvers{N}{\infty}\,0$$ et $$\int_0^1\dfrac{\sqrt{1-t^2}}{1+t}\ud t \int_0^1\dfrac{1-t}{\sqrt{1-t^2}}\ud t=\left[\Arcsin (t)+\sqrt{1-t^2}\right]_0^1=\dfrac{\pi}{2}-1$$ On en déduit alors que $\dsum_{n\geq 0}(-1)^na_n=\dfrac{\pi}{2-1}$.
  2. Soit $n\geq 2$, on a $$\begin{array}{lcl} a_n&=&\dsp \int_0^1 \dfrac{-t^{n-1}}{3}\left(-3t\sqrt{1-t^2}\right)\ud t = \int_0^1 \dfrac{-t^{n-1}}{3}\left((1-t^2)^{3/2}\right)'\ud t\\ &&\\ &=&\dsp \left[ \frac{-t^{n-1}}{3}(1-t^2)^{3/2}\right]_0^1+\dfrac{n-1}{3}\int_0^1t^{n-2}(1-t^2)^{3/2}\ud t\\ &&\\ &=&\dsp \dfrac{n-1}{3}\int_0^1t^{n-2}(1-t^2)\sqrt{1-t^2}\ud t =\dfrac{{n-1}}{3}\left(a_{n-2}-a_n\right). \end{array}$$ On en déduit alors, que pour tout $n\geq 2$, $a_n=\dfrac{n-1}{n+2}a_{n-2}$.
  3. Pour $n\geq 1$, on définit la suite $U_n$ par $U_n=n(n+1)(n+2)a_na_{n-1}$, ainsi on a $$U_{n+1}=(n+1)(n+2)(n+3)a_{n+1}a_n=(n+1)(n+2)(n+3)\dfrac{na_{n-1}}{n+3}a_n=U_n$$ Donc la suite $(U_n)$ est constante.
  4. Soit $n\geq 1$, on a $$a_n^2\leq a_na_{n-1}=\dfrac{a_1a_0}{n(n+1)(n+2)}\Longrightarrow 0\leq a_n\leq \dfrac{a_0a_1}{n^{3/2}}$$ La série $\dsum \dfrac{a_0a_1}{n^{3/2}}$ converge, il en va de même pour $\dsum a_n$.

($\star\star$) Correction

On pose $a_n=\dsum_{k=n}^\infty\frac{(-1)^k}{k}$. Montrer que la série $\dsum_{n\geq 1}a_n$ est convergente puis calculer sa somme.

Correction

On sait que la série $\dsum \dfrac{(-1)^k}{k}$ converge, donc $a_n$ est le reste d'ordre $n-1$ de cette série donc $a_n$ est bien définie de plus $a_n\tendversN 0$ ( oui mais ça ne montre pas la convergence de $\dsum a_n$).
Soit $n\in \N^*$, on a $$\left.\begin{array}{lclcl} a_n+a_{n+1}&=&\dsp\sum_{k=n}^\infty\dfrac{(-1)^k}{k}+\dsum_{k=n}^\infty\dfrac{(-1)^{k+1}}{k+1}&=&\dsum_{k\geq n}\dfrac{(-1)^k}{k(k+1)}\\ &&&&\\ a_n-a_{n+1}&=&\dfrac{(-1)^n}{n} \end{array} \right\}$$ donc $$\boxed{\textcolor{blue}{ a_n=\dfrac{(-1)^n}{2n}+\dsum_{k\geq n}\dfrac{(-1)^k}{2k(k+1)}}}.$$ Or la série de terme générale $\dfrac{(-1)^n}{2n}$ est une série convergente, et la série de terme générale $\dsum_{k\geq n}\dfrac{(-1)^k}{2k(k+1)}$ est une série convergente car $\dsum_{k\geq n}\dfrac{(-1)^k}{2k(k+1)}=\underset{n\rightarrow\infty}{\mathrm{O}}\left(\frac{1}{n^2}\right)$.
Donc la série $\dsum_{n\geq 1}a_n$ est convergente.
Soit $n\geq 2$, on a $$\begin{array}{lcl} a_1&=&-\ln(2)=\dsum_{k=0}^{n-1}\dfrac{(-1)^k}{k}+a_n=\dsum_{k=1}^{n-1}(-1)^k\int_0^1t^{k-1}\ud t+a_n\\ &&\\ &=&\dsp-\int_0^1\left(\dsum_{k=0}^{n-2}t^k\ud t\right)+a_n=-\int_0^1\dfrac{1-(-t)^{n-1}}{1+t}\ud t+a_n. \end{array}$$ Ce qui donne, pour tout $n\geq 2$, $a_n=-\dsp\int_0^1\dfrac{(-t)^{n-1}\ud t}{1+t}$.
Soit $N\geq 2$, on a $$\dsum_{n=1}^Na_n=a_1-\dsum_{n=2}^N\int_0^1 \dfrac{(-t)^{n-1}\ud t}{1+t}=a_1-\int_0^1\dsum_{n=1}^{N-1}\dfrac{(-t)^n\ud t}{1+t}=a_1+\int_0^1\dfrac{t\ud t}{(1+t)^2}+\int_0^1\dfrac{(-t)^N}{(1+t)^2}\ud t\quad\quad (\star).$$ D'autre part, $$\forall t\in [0,1[,\quad \abs{\dfrac{(-t)^N}{(1+t)^2}}\leq t^N\Longrightarrow \abs{\int_0^1\dfrac{(-t)^N}{(1+t)^2}\ud t}\leq \int_0^1t^N\ud t=\dfrac{1}{N+1}\tendvers{N}{\infty}\,0$$ Ensuite, un calcul simple donne $\dsp\int_0^1\dfrac{t\ud t}{(1+t)^2}= \int_0^1\dfrac{\ud t}{(1+t)}-\int_0^1\dfrac{\ud t}{(1+t)^2}=-a_1-\dfrac{1}{2}$ .
En remplaçant dans la relation $(\star)$, on trouve $$\boxed{ \dsum_{n\geq 1} a_n=\dfrac{-1}{2}}.$$

(MINES PC 2003) Correction

On pose $u_n=\ln\left(1+\dfrac{(-1)^n}{n^\alpha}\right)$ avec $\alpha>0$. Déterminer la nature de $\dsum u_n$.

Correction

On fait un DL $$u_n=\dfrac{(-1)^n}{n^\alpha}-\dfrac{1}{n^{2\alpha}} + \underset{n\to\infty}{\mathrm{o}}\left(\frac{1}{n^{2\alpha}}\right)= \dfrac{(-1)^n}{n^\alpha} +v_n \text{ avec } v_n \sim \dfrac{1}{n^{2\alpha}}.$$ Le premier terme converge par le CSSA. Le second terme converge pour $\alpha>\dfrac12$, et diverge sinon.
Ainsi, si $\alpha\in ]0,\dfrac12]$ : divergence; si $\alpha\in]\dfrac12,1]$ : semi-convergence; si $\alpha>1$ : convergence absolue.

(CCP PC 2001) Correction

On considère la série de terme général $\displaystyle u_n=\dfrac{(-1)^{n+1}}{n^2-2^2}$ pour $n\geq 3$. La série converge-t-elle? Si oui, en calculer la somme.

Correction

On commence par chercher $a$ et $b$ deux réels tels que $\dfrac{1}{n^2-2^2}=\dfrac{a}{n-2}+\dfrac{b}{n-2}$.
Après calculs, on trouve $$\dsp u_n=\dfrac{(-1)^{n+1}}{4}\left[\dfrac{1}{n-2}-\dfrac{1}{n+2}\right]=\dfrac{(-1)^{n+1}}{4}\left[\left(\dfrac{1}{n-2}-\dfrac{1}{n}\right)-\left(\dfrac{1}{n+2}-\frac{1}{n}\right)\right],$$ on obtient donc une série télescopique. Après calculs, on trouve $\boxed{\dsum_{n\geq 3}u_n=\dfrac{7}{48}}$.

(CCP PC 2017) Correction

Montrer la convergence de la série de terme général $u_n=\dfrac{(-1)^n}{2n+1}$.
Montrer que sa somme $S$ vérifie $\dfrac{2}{3}< S< 1$.
Calculer, pour $k\in \N$, $\dsp\int_0^1(-t)^k$ et exprimer la somme partielle $S_n$ à l'aide d'une intégrale. Retrouver la convergence de la série et calculer $S$.
Calculer $\dsp\int_0^1\dfrac{t^2\ud t}{(1+t^2)^2}$, on note $w_n$ le reste d'ordre $n$ de la série, montre que $\dsum w_n$ converge et calculer sa somme.

Correction

Il s'agit d'une série alternée, on applique le CSSA.
D'après le cours $S\in [u_0+u_1,u_0]$ (puisque $u_0>0$), on a $$u_0=1,\, u_0+u_1= 1-\frac{1}{3}=\dfrac{2}{3}$$ Voir la correction de l'exercice suivant.
$$\begin{array}{lcl} \dsp\int_0^1\dfrac{t^2\ud t}{(1+t^2)^2}&=&\dsp \dfrac{-1}{2}\int_0^1t\dfrac{-2t\ud t}{(1+t^2)^2}\\ &=&\dsp \left[\dfrac{-1}{2}t \dfrac{1}{1+t^2}\right]_0^1-\dfrac{-1}{2}\int_0^1\dfrac{\ud t}{(1+t^2)}\\ &=&\dsp \left[\dfrac{-t}{2} \dfrac{1}{1+t^2}\right]_0^1+\dfrac{1}{2}\left[\Arctan (t)\right]_0^1=\dfrac{\pi}{8}-\dfrac{1}{4} \end{array}$$ Soient $n\in \N,\,m\in \N$ , pour tout $x\in [0,1]$, on a: $$\begin{array}{lcl} \dfrac{x^{2n+2}}{1+x^2}&=&\dfrac{x^{2n+2}}{1-(-x^2)}=x^{2n+2}\dsum_{k=0}^m(-x^2)^k-x^{2n+2}\dfrac{(-x^2)^{m+1}}{1+x^2}\\ &=&(-1)^{n+1}\dsum_{k=n+1}^{n+m+1}(-1)^kx^{2k}+ \dfrac{(-1)^{m}x^{2(n+m+2)}}{1+x^2}. \end{array}$$ En intégrant la relation précédente entre $0$ et $1$, on trouve: $$\begin{array}{lcl} \dsp (-1)^{n+1}\int_0^1\dfrac{x^{n+2}\ud x}{1+x^2}&=&\dsp \dsum_{k=n+1}^{n+m+1}\int_0^1(-x)^k\ud x+(-1)^{n+1}\int_0^1\dfrac{(-1)^{m}x^{2(n+m+2)}}{1+x^2}\ud x\\ &=&\dsp \dsum_{k=n+1}^{n+m+1}\dfrac{(-1)^{k}}{2k+1}+(-1)^{n+1}\int_0^1\dfrac{(-1)^{m}x^{2n+2m+4}}{1+x^2}\ud x. \end{array}$$ Ainsi, pour tout $m\geq n$, on a $$\abs{(-1)^{n+1}\int_0^1\dfrac{x^{2n+2}\ud x}{1+x^2}-\dsum_{k=n+1}^{n+m+1}\dfrac{(-1)^{k}}{2k+1}}\leq \int_0^1x^{2n+2m+4}\ud x\tendvers{m}{\infty}0$$ Donc $\left(\dsum_{k=n+1}^{m}\frac{(-1)^k}{2k+1}\right)_{m\geq n+1}$ converge vers $(-1)^{n+1}\dsp\int_0^1\dfrac{x^{2n+2}\ud x}{1+x^2}$, d'autre part cette suite converge également vers $w_n$, donc par unicité de la limite, on trouve $$\boxed{ w_n=(-1)^{n+1}\int_0^1\dfrac{x^{2n+2}\ud x}{1+x^2}}.$$ Soit $n\geq 1$, on a $$\begin{array}{lcl} \dsum_{k=0}^nw_k&=&\dsp \dsum_{k=0}^n(-1)^{k+1}\int_0^1\dfrac{x^{2k+2}\ud x}{1+x^2}\\ &&\\ &=&\dsp -\int_0^1\left(\dsum_{k=0}^n(-x^2)^k\right)\dfrac{x^2\ud x}{1+x^2}= -\int_0^1\dfrac{1-(-x^2)^{n+1}}{1+x^2}\dfrac{x^2\ud x}{1+x^2}\\ &&\\ &=&\dsp -\int_0^1\dfrac{x^2\ud x}{(1+x^2)^2}+(-1)^{n+2}\int_0^1\dfrac{x^{2n+4}\ud x}{(1+x^2)^2} \tendversN-\int_0^1\dfrac{\ud x^2}{(1+x^2)^2} \end{array}$$ On en déduit alors $$\boxed{\dsum_{n\geq 0}w_n=-\int_0^1\dfrac{x^2\ud x}{(1+x^2)^2}=\dfrac{1}{4}-\dfrac{\pi}{8}}.$$

(EIVP 2017) Correction

Soit $n\in \N^*$, on pose $u_n=\dsum_{k\geq 0}\dfrac{(-1)^k}{n+1+k}$.

  1. Justifier l'existence de $u_n$, puis étudier la convergence de $\dsum u_n$ (on pourra étudier $u_n+u_{n+1}$).
  2. Convergence de $\dsum (-1)^nu_n$.

Correction

  1. Soit $n\in\N^*$, la suite $\left(\frac{1}{n+1+k}\right)_{k\in \N}$ est strictement décroissante, de plus $\frac{1}{n+1+k}\tendvers{k}{\infty}\,0$. Donc d'après CSSA $\dsum_{k\geq 0}\dfrac{(-1)^k}{n+1+k}$ converge, autrement dit, $u_n$ est bien définie.
    En utilisant l'indication donnée dans la question, on a pour tout $n\in \N$, $$u_n+u_{n+1}=\dsum_{k\geq 0}\dfrac{(-1)^k}{n+1+k}+\dsum_{k\geq 0}\dfrac{(-1)^k}{n+2+k}=\dsum_{k\geq 0}\dfrac{(-1)^k}{n+1+k}+\dsum_{k\geq 1}\dfrac{(-1)^{k-1}}{n+1+k}=\dfrac{1}{n+1}.$$ On en déduit que $\dsum u_n$ diverge, sinon, on aurait $\dsum u_{n+1}$ CV donc $\dsum u_n+u_{n+1}$ CV ce qui est impossible.
  2. D'après le cours, le signe de $u_n$ est le même que le premier terme de $\dsum_{k\geq 0}\dfrac{(-1)^k}{n+1+k}$ i.e. $u_n>0$. De plus $u_n\tendversN\,0$ (car $u_n\leq \dfrac{1}{n+1}$).
    Montrons que $(u_n)$ est décroissante, soit $n\in \N^*$, $$u_n-u_{n+1}=\dsum_{k\geq 0}\dfrac{(-1)^k}{n+1+k}-\dsum_{k\geq 0}\dfrac{(-1)^k}{n+2+k}=\dsum_{k\geq 0}\dfrac{(-1)^k}{(n+1+k)(n+2+k)},$$ Il s'agit encore d'une série alternée, donc le signe de la somme est le même que son premier terme $\dfrac{(-1)^0}{(n+1)(n+2)}>0$, i.e. $u_n-u_{n+1}>0$.
    Conclusion, la suite $(u_n)$ est décroissante et tend vers $0$, alors d'après CSSA $\dsum (-1)^nu_n$ CV.

($\star\star$) Correction

On considère les suites $(h_n)_{n\geq 1}$ et $(u_n)_{n\geq 1}$ définies par $h_n =\dsum_{k=1}^n\dfrac{1}{k}$ et $u_n = h_n- \ln( n)$.

  1. En étudiant la série de terme général $u_{n+1}- u_n$, montrer que la suite $(u_n)_{n\geq 1}$ est convergente. On note $\gamma$ sa limite.
  2. Justifier le fait que $h_n = \ln( n) + \gamma + \underset{n\to\infty}{\mathrm{o}}(1)$.
    Montrer qu'il existe deux réels $a$ et $b$ que l'on déterminera, tels que, $\dsp \forall n\in \N^*,\,\dsum_{k=1}^{2n}\dfrac{(-1)^k}{k}= ah_{2n}- bh_n$.
    En déduire $\dsum_{k\geq 1}\dfrac{(-1)^k}{k}=-\ln(2)$.
  3. On considère $\alpha$ dans $\R$. Soit $(w_n)_{n\geq 1}$ la suite définie par $w_n = \frac{-\alpha}{n}$ si $n$ est un multiple de $4$ et $w_n = \frac{1}{n}$ sinon. Pour $n \geq 1 $ on pose $S_n =\dsum_{k=1}^nw_k$.
    Montrer que la suite $(S_{4n})_{n\geq 1}$ est convergente si et seulement si $\alpha = 3$.
    On suppose que $\alpha = 3$. Établir la convergence de la série $\dsum w_n$ et calculer sa somme
Correction

  1. Soit $n\in\N^*$, on a $$\begin{array}{lcl} u_{n+1}-u_n&=& h_{n+1}-h_n-\ln(n+1)-\ln(n)=\dfrac{1}{n+1}-\ln\left(1+\frac{1}{n}\right)\\ &=&\dfrac{1}{n+1}-\dfrac{1}{n}+\underset{n\to\infty}{\mathrm{O}}\left(\frac{1}{n^2}\right) =\underset{n\to\infty}{\mathrm{O}}\left(\frac{1}{n^2}\right). \end{array}$$ On en déduit que $\dsum (u_{n+1}-u_n)$ converge d'où la convergence de la suite $(u_n)$.
  2. Comme $u_n=h_n-\ln(n)$ et en utilisant la question précédente, on trouve: $h_n=\ln(n)+\dsp\lim_{n\to \infty} u_n +\underset{n\to\infty}{\mathrm{o}}(1)$.
    Soit $n\geq 1$, on a $$\begin{array}{lcl} \dsum_{k=1}^{2n}\dfrac{(-1)^k}{k}&=& \dsum_{k=0}^{n-1}\dfrac{-1}{2k+1}+ \dsum_{k=1}^{n}\dfrac{1}{2k}\\ &&\\ &=&\dsum_{k=0}^{n-1}\dfrac{-1}{2k+1}-\dsum_{k=1}^{n}\dfrac{1}{2k}+ \dsum_{k=1}^{n}\dfrac{1}{k}\\ &&\\ &=&\dsum_{k=}^{2n}\dfrac{-1}{k}+ \dsum_{k=1}^{n}\dfrac{1}{k}=-h_{2n}+h_n \end{array}$$ En particulier, pour $n$ assez grand, on a $$\dsum_{k=1}^{2n}\dfrac{(-1)^k}{k} =-\ln(2n)+\ln(n)+\underset{n\to\infty}{\mathrm{o}}\left(1\right)$$ Donc $\dsum_{k=1}^{2n}\dfrac{(-1)^k}{k}\tendversN\,-\ln(2)$, la série $\dsum \dfrac{(-1)^k}{k}$ étant convergente donc la limite de la somme partielle d'ordre $2n$ est la somme de la série. Conclusion $$\boxed{\dsum_{k\geq 1} \dfrac{(-1)^k}{k}=-\ln(2)}.$$
  3. Soit $n\geq 1$, on a $$\begin{array}{lcl} S_{4n}&=&\dsum_{k=1}^{4n}w_k=\dsum_{k=0}^n\left(\dfrac{1}{4k+1}+\dfrac{1}{4k+2}+\dfrac{1}{4k+3}+\dfrac{-\alpha}{4k+4}\right)\\ &&\\ &=&2\dsum_{k=0}^n\dfrac{(96-32\alpha)k^3+(192-48\alpha)k^2+(118-22\alpha)k+22-3\alpha}{(4k+1)(4k+2)(4k+3)(4k+4)} \end{array}$$ Si $\alpha\neq 3$ alors $$\dfrac{(96-32\alpha)k^3+(192-48\alpha)k^2+(118-22\alpha)k+22-3\alpha}{(4k+1)(4k+2)(4k+3)(4k+4)} \thicksim \dfrac{96-3\alpha}{256 k}$$ donc $S_{4n}$ diverge.
    Si $\alpha=3$ alors $$\dfrac{(96-32\alpha)k^3+(192-48\alpha)k^2+(118-22\alpha)k+22-3\alpha}{(4k+1)(4k+2)(4k+3)(4k+4)} \thicksim \dfrac{3}{16 k^2}$$ Dans ce cas, $S_{4n}$ tend vers une limite finie car $\sum \dfrac{1}{k^2}$ converge.
    Soit $N\in \N^*$, et $n\in \N$ tel que $N=4n+r$ (avec $r\in \inter{0,3}$), alors $$S_N=\dsum_{k=1}^Nw_k =S_{4n}+\dsum_{k=1}^r\dfrac{1}{4n+k}\Longrightarrow\abs{S_N-S_{4n}}\leq \dfrac{3}{4n+1}\tendvers{N}{\infty}0$$ Ce qui prouve que la série $\dsum w_n$ converge.
    Soit $n\geq 1$, on a $$\begin{array}{lcl} \dsum_{k=1}^{4n}w_k&=& \dsum_{k=0}^{n-1}\dfrac{1}{4k+1}+ \dsum_{k=0}^{n-1}\dfrac{1}{4k+2}+\dsum_{k=0}^{n-1}\dfrac{1}{4k+3}-\dsum_{k=1}^{n}\dfrac{3}{4k}\\ &&\\ &=&\dsum_{k=0}^{n-1}\dfrac{1}{4k+1}+ \dsum_{k=0}^{n-1}\dfrac{1}{4k+2}+\dsum_{k=0}^{n-1}\dfrac{1}{4k+3}+\dsum_{k=0}^{n-1}\dfrac{1}{4k+4}-\dsum_{k=1}^{n}\dfrac{4}{4k}\\ &&\\ &=&\dsum_{k=}^{4n}\dfrac{1}{k}- \dsum_{k=1}^{n}\dfrac{1}{k}=h_{4n}-h_n \end{array}$$ Donc $S_{4n}\tendversN\, \ln(4)$, ce qui donne $\dsum w_k=\ln(4)$.

($\star\star$) Correction

Soit $f$ une fonction de $\R_+^*$ dans $\R$ dérivable telle que $f'$ est monotone et $\limiteX{x}{\infty}f'(x)=0$.

  1. On se donne deux suites $(y_k)$ et $(x_k)$ telles que: $\forall k\in \N,\,x_k\in ]k,k+1/2[,\,\,y_k\in ]k+1/2,k+1[$.
    Montrer que la série $\dsum_{n\geq 0} f'(y_n)-f'(x_n)$ converge.
  2. Montrer la convergence de la suite $(u_n)_{n\geq 1}$ définie par: $$ u_n=\dfrac{f(1)}{2}+\dfrac{f(n)}{2}+\dsum_{k=2}^{n-1}f(k)-\dsp\int_1^nf(t)\ud t.$$
  3. Application:
    1. Appliquer le résultat pour $f(x)=\dfrac{1}{x}$
    2. Même chose pour $f(x)=\ln(x)$ (lien avec la formule de Stirling?).

Correction

  1. Quitte à multiplier $f$ par $-1$, on peut supposer que $f'>0$, puisque $\limiteX{x}{\infty}f'(x)=0$ alors $f'$ est décroissante. Ainsi, $$\forall k\geq 1,\quad f'(k+1)\leq f'(y_k)\leq f'(k+1/2)\leq f'(x_k)\leq f'(x_k).$$ Ce qui donne, $$\forall k\geq 1,\quad f'(k+1)-f'(k)\leq f'(y_k)-f'(x_k)\leq 0.$$ Comme la série télescopique $\dsum f'(k+1)-f'(k)$ converge puisque $f'(n)\tendversN\,0$ alors $\dsum_{n\geq 0} f'(y_n)-f'(x_n)$ converge.
  2. Pour $t\geq 1$, on pose $F(t)=\dsp\int_1^tf(u)\ud u$. Alors $F\in \CC^2([1,\infty[)$. En utilisant la formule de Taylor-Lagrange, on a pour tout $k\geq1$, $$\exists x_k\in ]k,k+1/2[,\, F(k+1/2)=F(k)+(k+1/2-k)F'(k)+\dfrac{(k+1/2-k)^2}{2}F''(x_k)=F(k)+\dfrac{1}{2}f(k)+\dfrac{1}{8}f'(x_k).$$ $$ \exists y_k\in ]k+1/2,k+1[,\, F(k+1/2)=F(k+1)+(k+1/2-k-1)F'(k)+\dfrac{(k+1/2-k-1)^2}{2}F''(y_k)=F(k+1)-\dfrac{1}{2}f(k+1)+\dfrac{1}{8}f'(y_k).$$ Ainsi, $$0 = F(k)-F(k+1)+\dfrac{1}{2}(f(k)+f(k+1))-\dfrac{1}{8}(f(y_k)-f(x_k))= -\int_k^{k+1}f(t)\ud t+\dfrac{1}{2}(f(k)+f(k+1))-\dfrac{1}{8}(f(y_k)-f(x_k)).$$ En faisant la somme, on obtient, $$0 =-\int_1^nf(t)\ud t +\dfrac{f(1)+f(n)}{2}+\dsum_{k=2}^{n-1}f(k)-\dfrac{1}{8}\dsum_{k=1}^n(f'(y_k)-f'(x_k))\Longrightarrow u_n=\dfrac{1}{8}\dsum_{k=1}^n(f'(y_k)-f'(x_k)).$$ D'après la question précédente, $\dsum_{k=1}^nf'(y_k)-f'(x_k)$ converge, donc la suite $(u_n)$ admet une limite finie.
    1. La fonction $f(x)=\dfrac{1}{x}$ vérifie les hypothèse, donc $u_n$ tend vers une limite finie, or dans ce cas, $$u_n=\dfrac{1}{2}+\dfrac{1}{2n}+\dsum_{k=2}^{n-1}\dfrac{1}{k}-\int_1^n\dfrac{\ud x}{x}=\dsum_{k=1}^n\dfrac{1}{k}-\ln(n)-\dfrac{1}{2}-\dfrac{1}{2n}$$ Notons $\ell$ la limite de la suite $(u_n)$, on en déduit alors que $$\dsum_{k=1}^n\dfrac{1}{k}-\ln(n)\tendversN\, \ell+\dfrac{1}{2}.$$ Autrement dit, $\dsum_{k=1}^n\dfrac{1}{k}\underset{n\to \infty}{\thicksim}\ln(n).$
    2. La fonction $f(x)=\ln(x)$ vérifie les hypothèse, et $$u_n=\dsum_{k=2}^n\ln(k)-\dfrac{\ln(n)}{2}-n\ln(n)+n=\ln(n!)-\ln(\sqrt{n})-\ln\left(\left(\dfrac{n}{\ee}\right)^n\right)\tendversN\,\ell$$ On en déduit alors, qu'il existe $K>0$ tel que $n!\underset{n\to \infty}{\thicksim}K\sqrt{n}\left(\frac{n}{\ee}\right)^n.$
(E3A 200?) Correction

Étude de $R_n=\dsum_{k\geq n+1}\dfrac{(-1)^n}{n}$

  1. Montrer que la série $\dsum_{n\geq 1}\dfrac{(-1)^n}{n}$ converge, et donner une majoration de $R_n$ ($n\in \N$).
  2. A l'aide d'une suit géométrique, montrer que, pour tout $n\geq 0$, $\dsp R_n=(-1)^{n+1}\int_0^1\dfrac{x^n\ud x}{1+x}$.
  3. Par une IPP, montrer que $R_n=K\dfrac{(-1)^{n+1}}{n^{\beta}}+\underset{n\to\infty}{\mathrm{O}}\left(\frac{1}{n^{\beta+1}}\right)$ avec $K\in \R^*,\,\beta\in \N^*$.
  4. En déduire la nature de la série de terme général $R_n$, puis calculer sa somme.
Correction

$\mathbf{1. }$ La série $\dsum\dfrac{(-1)^n}{n}$ est alternée. De plus la suite $(\frac{1}{n})_{n\geq 1}$ est strictement décroissante et tend vers $0$. Donc d'après le CSSA, la série $\dsum\dfrac{(-1)^n}{n}$ converge, de plus, on a $$\forall n\in \N^*,\quad \abs{R_n}\leq \dfrac{1}{n+1}.$$ $\mathbf{2. }$ Soient $n\in \N,\,m\in \N$ tel que $m\geq n$, pour tout $x\in [0,1]$, on a: $$\dfrac{x^n}{1+x}=\dfrac{x^n}{1-(-x)}=x^n\dsum_{k=0}^m(-x)^k+x^n\dfrac{(-x)^{m+1}}{1+x}=(-1)^n\dsum_{k=n}^{n+m}(-1)^kx^k-\dfrac{(-1)^{m}x^{n+m}}{1+x}.$$ En intégrant la relation précédente entre $0$ et $1$, on trouve: $$\begin{array}{lcl} (-1)^{n+1}\int_0^1\dfrac{x^n\ud x}{1+x}&=&-\dsum_{k=n}^{n+m}\int_0^1(-x)^k\ud x-\int_0^1\dfrac{(-1)^{m}x^{n+m}}{1+x}\ud x\\ &=&\dsum_{k=n}^{n+m}\dfrac{(-1)^{k+1}}{k+1}-\int_0^1\dfrac{(-1)^{m}x^{n+m}}{1+x}\ud x. \end{array}$$ Ainsi, pour tout $m\geq n$, on a $$\abs{(-1)^{n+1}\int_0^1\dfrac{x^n\ud x}{1+x}-\dsum_{k=n+1}^{n+m+1}\dfrac{(-1)^{k}}{k}}\leq \abs{\int_0^1\dfrac{(-1)^{m}x^{n+m}}{1+x}\ud x}\leq \int_0^1x^{n+m}\ud x\tendvers{m}{\infty}0$$ Donc $\left(\dsum_{k=n+1}^{m}\frac{(-1)^k}{k}\right)_{m\geq n+1}$ converge vers $(-1)^{n+1}\dsp\int_0^1\dfrac{x^n\ud x}{1+x}$, d'autre part cette suite converge également vers $R_n$, donc par unicité de la limite, on trouve $$\boxed{ R_n=(-1)^{n+1}\int_0^1\dfrac{x^n\ud x}{1+x}}.$$ $\mathbf{3. }$ Soit $n\in \N^*$, on a $$\begin{array}{lcl} (-1)^{n+1}R_n&=&\dsp \int_0^1\dfrac{x^n\ud x}{1+x}\\ &=&\dsp \int_0^1x^{n-1}\dfrac{x}{1+x}\ud x=\left[\dfrac{x^n}{n}\dfrac{x}{1+x}\right]_0^1-\int_0^1\dfrac{x^n}{n}\dfrac{1}{(1+x)^2}\ud x =\dfrac{1}{2n}+I_n. \end{array}$$ Avec $$\abs{I_n}=\int_0^1\dfrac{x^n}{n(1+x)^2}\ud x\leq \dfrac{1}{n}\int_0^1x^n\ud x=\dfrac{1}{2n(n+1)}\Longrightarrow I_n=\underset{n\to\infty}{\mathrm{O}}\left(\frac{1}{n^2}\right).$$ On en déduit $$\boxed{R_n=\dfrac{(-1)^{n+1}}{2n}+ \underset{n\to\infty}{\mathrm{O}}\left(\frac{1}{n^2}\right)}.$$ Donc on peut prendre $\beta =1$ et $K=\dfrac{1}{2}$.
$\mathbf{4. }$ D'après la question précédente, on a pour $n\geq 1$, $R_n=\dfrac{(-1)^{n+1}}{2n}+w_n$ avec $w_n=\underset{n\to\infty}{\mathrm{O}}\left(\frac{1}{n^2}\right)$, donc $R_n$ est la somme de deux termes généraux des séries convergentes, donc $\dsum R_n$ converge.
Soit $n\geq 1$, on a $$\begin{array}{lcl} \dsum_{k=0}^nR_k&=&\dsum_{k=0}^n(-1)^{k+1}\int_0^1\dfrac{x^k\ud x}{1+x}=-\int_0^1\left(\dsum_{k=0}^n(-x)^k\right)\dfrac{\ud x}{1+x}=-\int_0^1\dfrac{1-(-x)^{n+1}}{1+x}\dfrac{\ud x}{1+x}\\ &&\\ &=&\dsp -\int_0^1\dfrac{\ud x}{(1+x)^2}+(-1)^{n+1}\int_0^1\dfrac{x^{n+1}\ud x}{(1+x)^2}\tendversN-\int_0^1\dfrac{\ud x}{(1+x)^2} \end{array}$$ On en déduit alors $\boxed{\dsum_{n\geq 0}R_n=-\left[\dfrac{-1}{(1+x)}\right]_0^1=\dfrac{-1}{2}}$.

($\star\star$) Correction

Soit $x\in [0,1]$, on note $S(x)=\dsum_{n=0}^\infty\dfrac{(-1)^n x^{2n+1}}{2n+1}$.

  1. Montrer que $S(x)$ converge et que $S(x)=\Arctan(x)$.
  2. Justifier la relation suivante: \begin{equation} %\label{Eq1} \pi=4\dsum_{k=0}^\infty \dfrac{(-1)^{k}}{2k+1} \end{equation} Pour $n\in \N^*$. Donner une majoration de $\abs{\pi -4 \dsum_{k=0}^n \dfrac{(-1)^{k}}{2k+1}}$, en déduire la plus petite valeur de $n$ pour qu'on ait $\dsp\abs{\pi -4 \dsum_{k=0}^n \dfrac{(-1)^{k}}{2k+1}}\leq 10^{-6}.$
  3. Justifier la relation suivante: \begin{equation} %\label{Eq2} \pi=4\dsum_{k=0}^\infty \dfrac{(-1)^{k}}{2k+1}\left(\frac{1}{2^{2k+1}}+\frac{1}{3^{2k+1}}\right) \end{equation} Pour $n\in \N^*$. Donner une majoration de $\abs{\pi -4 \dsum_{k=0}^n \dfrac{(-1)^{k}}{2k+1}\left(\frac{1}{2^{2k+1}}+\frac{1}{3^{2k+1}}\right)}$, en déduire la plus petite valeur de $n$ pour qu'on ait $\dsp\abs{\pi -4 \dsum_{k=0}^n \dfrac{(-1)^{k}}{2k+1}\left(\frac{1}{2^{2k+1}}+\frac{1}{3^{2k+1}}\right)}\leq 10^{-6}.$

Correction

Soit $x\in [0,1]$ la série $S(x)$ vérifie les conditions de théorème CSSA, donc $S(x) $ converge. (Pour $x\in [0,1[$, $S(x)$ est absolument convergente).
Soit $n\geq 1$, on a $$\begin{array}{lcl} S_n(x)&=&\dsum_{k=0}^n\dfrac{(-1)^kx^{2k+1}}{2k+1}=\dsum_{k=0}^n(-1)^k\int_0^xt^{2k}\ud t =\dsp \int_0^x\left(\sum_{k=0}^n(-t^2)^{k}\right)\ud t\\ &=&\dsp \int_0^x\dfrac{1-(-t^2)^{n+1}}{1+t^2}\ud t=\int_0^x\dfrac{1}{1+t^2}\ud t-\int_0^x\dfrac{(-t^2)^{n+1}}{1+t^2}\ud t=\Arctan(x)+ \int_0^x\dfrac{(-t^2)^n}{1+t^2}\ud t \end{array}$$ On en déduit alors, $$\abs{S_n(x)-\Arctan(x)}\leq \abs{\int_0^x\dfrac{(-t^2)^{n+1}}{1+t^2}\ud t}\leq \int_0^xt^{2n+2}\ud t=\dfrac{x^{2n+3}}{2n+3}\tendversN\,0$$ Ce qui prouve que $S(x)$ est convergente, de plus, $$\boxed{\forall x\in [0,1],\quad \dsum_{n\geq 0}\dfrac{(-1)^nx^{2n+1}}{2n+1} =\Arctan(x)}.$$ En particulier, pour $x=1$, on trouve $\dsum_{n\geq 0}\dfrac{(-1)^n}{2n+1} =\Arctan(1)=\dfrac{\pi}{4}$.
En utilisant le théorème de majoration, on trouve: $$\forall n\geq 0,\quad\abs{\dfrac{\pi}{4}-\dsum_{k=0}^n\dfrac{(-1)^n}{2k+1}}\leq \dfrac{1}{2n+3}\Longrightarrow \abs{\pi -4 \dsum_{k=0}^n \dfrac{(-1)^{k}}{2k+1}}\leq\dfrac{4}{2n+3}$$ Donc si on souhaite une approximation de $\pi$ en utilisant cette relation à $10^{-6}$ il faut prendre $\textcolor{red}{n\geq n_0=1999999}$.
Pour montrer la relation \ref{Eq2}, il suffit de remarquer que $$\Arctan (1/2)+\Arctan(1/3)=\Arctan\left(\dfrac{\frac{1}{2}+\frac{1}{3}}{1-\frac{1}{2}\frac{1}{3}}\right)=\Arctan\left(\dfrac{\frac{5}{6}}{1-\frac{1}{6}}\right) =\Arctan (1)=\dfrac{\pi}{4},$$ puisque $1/2,\,1/3\in [0,1]$, on utilise alors la fonction $S$, ce qui donne, $$\dfrac{\pi}{4}=\dsum_{k\geq 0}\dfrac{(-1)^k}{(2k+1)2^{2k+1}}+\dsum_{k\geq 0}\dfrac{(-1)^k}{(2k+1)3^{2k+1}}\Longrightarrow \pi= 4\dsum_{k=0}^\infty \dfrac{(-1)^{k}}{2k+1}\left(\frac{1}{2^{2k+1}}+\frac{1}{3^{2k+1}}\right).$$ En utilisant le théorème de majoration du reste, on trouve: $$\abs{\pi -4 \dsum_{k=0}^n \dfrac{(-1)^{k}}{2k+1}\left(\frac{1}{2^{2k+1}}+\frac{1}{3^{2k+1}}\right)}\leq \dfrac{4}{2n+3}\left(\dfrac{1}{2^{2n+3}}+\dfrac{1}{3^{2n+3}}\right).$$ Pour obtenir une erreur inférieur à $10^{-6}$ il faut prendre $n\geq n_1=8$.

(Contre exemple) Correction

On considère la suite $(u_n)_{n\geq 1}$ définie par $u_n=\dfrac{1}{n}$ si $n$ est paire, $\dfrac{-1}{n^2}$ sinon.
Étudier la convergence de $\dsum u_n$.

Correction

On a $(-1)^nu_n>0$ et $u_n\tendversN\,0$ puisque $\abs{u_n}\leq \dfrac{1}{n}$.
Soit $n>0$, on a: $$S_{2n}=\dsum_{k=1}^{2n}u_k=\dsum_{k=0}^{n-1}u_{2k+1}+\dsum_{k=1}^nu_{2k}= -\dsum_{k=0}^{n-1}\dfrac{1}{(2k+1)^2}+\dsum_{k=1}^n\dfrac{1}{2k}=-V_n+\dfrac{1}{2}H_n$$ On sait déjà que $H_n\tendversN\,\infty$, d'autre part, $$\forall n\geq 2,\quad V_n\leq 1+\dsum_{k=1}^n\dfrac{1}{4k^2}\leq 1+ \dfrac{1}{4}\dsum_{k\in \N^*}\dfrac{1}{k^2}=1+\dfrac{\pi^2}{6}$$ Ce qui implique que la suite $(V_n)$ est bornée. On en déduit alors que $S_{2n}\tendversN\,\infty$ ce qui implique $\dsum u_n $ diverge.

Séries télescopiques

($\star\star$) Correction

Montrer la convergence des séries suivantes, puis calculer leur somme ($m\in \N^*$):

  1. $\dsum_{n\geq 1}\dfrac{2n+1}{n^2(n+1)^2}$
  2. $\dsum_{n\geq 1}\dfrac{n}{(2n-1)^2(2n+1)^2}$
  3. $\dsum_{n\geq 1}\dfrac{1}{4n^2-1}$
  4. $\dsum_{n\geq 1}\dfrac{1}{n(n+1)\cdots(n+m)}$
  5. $\dsum_{n\geq 1}\dfrac{n-\sqrt{n^2-1}}{\sqrt{n(n+1)}}$
  6. $\dsum_{n\geq 1}\ln\left(\frac{n(2n+1)}{(n+1)(2n-1)}\right)$
  7. $\dsum_{n\geq 1}\dfrac{1}{(\sqrt{n}+\sqrt{n+1})\sqrt{n(n+1)}}$
  8. $\dsum_{n\geq 1}\dfrac{1}{n(n+m)}$
  9. $\dsum_{n\geq 1}\dfrac{n^2}{(n+1)(n+2)(n+3)(n+4)}$
Correction

  1. $\dsum_{n\geq 1}\dfrac{2n+1}{n^2(n+1)^2}$
    On écrit pour $n\geq 1,\,\,\dfrac{2n+1}{n^2(n+1)^2}=\dfrac{1}{n^2}-\dfrac{1}{(n+1)^2}$.
  2. $\dsum_{n\geq 1}\dfrac{n}{(2n-1)^2(2n+1)^2}$
    On écrit pour $n\geq 1,\, \dfrac{n}{(2n-1)^2(2n+1)^2}=\dfrac{1}{8}\left(\dfrac{1}{(2n-1)^2}-\dfrac{1}{(2n+1)^2}\right)$.
  3. $\dsum_{n\geq 1}\dfrac{1}{(\sqrt{n}+\sqrt{n+1})\sqrt{n(n+1)}}$.
    On écrit: $\dfrac{1}{(\sqrt{n}+\sqrt{n+1})\sqrt{n(n+1)}}=\dfrac{\sqrt{n+1}-\sqrt{n}}{\sqrt{n(n+1)}}=\dfrac{1}{\sqrt{n}}-\dfrac{1}{\sqrt{n+1}}$.
  4. $\dsum_{n\geq 1}\dfrac{1}{n(n+1)\cdots(n+m)}$
    On écrit, pour $n\geq 1$, $$\dfrac{1}{n(n+1)\cdots(n+m)}=\dfrac{1}{m}\left(\dfrac{1}{n(n+1)\cdots(n+m-1)}-\dfrac{1}{(n+1)\cdots(n+m)}\right)$$ $$\Longrightarrow \boxed{\dsum_{n\geq 1}\dfrac{1}{n(n+1)\cdots(n+m)}=\dfrac{1}{m\,m!}} \,\,(\,\star).$$
  5. $\dsum_{n\geq 1}\dfrac{n-\sqrt{n^2-1}}{\sqrt{n(n+1)}}$
    On écrit $\dfrac{n-\sqrt{n^2-1}}{\sqrt{n(n+1)}}=\dfrac{n-\sqrt{(n+1)(n-1)}}{\sqrt{n(n+1)}} =\dfrac{\sqrt{n}}{\sqrt{(n+1)}}-\dfrac{\sqrt{n-1}}{\sqrt{n}}$.
  6. $\dsum_{n\geq 1}\ln\left(\frac{n(2n+1)}{(n+1)(2n-1)}\right)$
    On a $\ln\left(\frac{n(2n+1)}{(n+1)(2n-1)}\right)=(\ln(n)-\ln(n+1))+(\ln(2n+1)-\ln(2n-1))$.
  7. $\dsum_{n\geq 1}\dfrac{1}{4n^2-1}$
    On écrit $\dfrac{1}{4n^2-1}=\dfrac{1/2}{2n-1}-\dfrac{1/2}{2n+1}$
  8. $\dsum_{n\geq 1}\dfrac{1}{n(n+m)}$
    On écrit $\dfrac{1}{n(n+m)}=\dfrac{1/m}{n}-\dfrac{1/m}{n+m}$.
  9. $\dsum_{n\geq 1}\dfrac{n^2}{(n+1)(n+2)(n+3)(n+4)}$
    En écrivant $n^2=n^2-1+1=(n-1)(n+1)-1$, on obtient, $$\begin{array}{ll} \dfrac{n^2}{(n+1)(n+2)(n+3)(n+4)}&=\dfrac{n-1}{(n+2)(n+3)(n+4)}\\ &\phantom{=}+\dfrac{1}{(n+1)(n+2)(n+3)(n+4)}\\ &\\ &=\dfrac{1}{(n+3)(n+4)}-\dfrac{3}{(n+2)(n+3)(n+4)}\\ &\phantom{=}\,-\dfrac{1}{(n+1)(n+2)(n+3)(n+4)} \end{array} $$ En utilisant la relation $(\star)$, on trouve: $$ \begin{array}{lcl} \dsum_{n\geq 1} \dfrac{1}{(n+3)(n+4)}&=&\dsum_{n\geq 4}\dfrac{1}{n(n+1)}\\ &=&\dfrac{1}{1\times 1!}-\dfrac{1}{2}-\dfrac{1}{6}-\dfrac{1}{12}=\dfrac{3}{12}\\ \dsum_{n\geq 1} \dfrac{1}{(n+2)(n+3)(n+4)}&=&\dsum_{n\geq 3}\dfrac{1}{n(n+1)(n+2)}\\ &=&\dfrac{1}{2\times 2!}-\dfrac{1}{6}-\dfrac{1}{24}=\dfrac{1}{24}\\ \dsum_{n\geq 1} \dfrac{1}{(n+1)(n+2)(n+3)(n+4)}&=&\dsum_{n\geq 2}\dfrac{1}{n(n+1)(n+2)(n+3)}\\ &=&\dfrac{1}{3\times 3!}-\dfrac{1}{24}=\dfrac{1}{72} \end{array} $$ Ce qui donne finalement, $$\boxed{\dsum_{n\geq 1}\dfrac{n^2}{(n+1)(n+2)(n+3)(n+4)} = \dfrac{3}{12}-\dfrac{3}{24}+\dfrac{1}{72}=\dfrac{5}{36}}.$$

(CCP PSI) Correction

On considère la suite $(u_n)_{n\in \N^*}$ définie par: $$u_1=1,\quad\quad\forall n\geq 0,\,u_{n+1}=1+u_1u_2\cdots u_n=1+\prod_{k= 1}^nu_k.$$

  1. Montrer que $u_n\tendversN \infty$.
  2. Montrer que la série $\dsum\frac{1}{u_n}$ converge et a pour somme $2$.
Correction

  1. On peut montrer par récurrence sur $n\in \N$, que $u_n\geq n$.
  2. Il suffit de remarque, que pour $n\geq 2$, on a $$u_n>1,\quad u_{n+1}=1+u_1u_2\cdots u_n=1+(u_n-1)u_n\Longrightarrow \dfrac{1}{u_{n+1}-1}=\dfrac{1}{u_n(u_n-1)}=\dfrac{1}{u_n-1}-\dfrac{1}{u_n}.$$ Ce qui donne $$\dfrac{1}{u_n}=\dfrac{1}{u_n-1}-\dfrac{1}{u_{n+1}-1}\Longrightarrow \dsum_{k=2}^n\dfrac{1}{u_k}=\dfrac{1}{u_2-1}-\dfrac{1}{u_{n+1}-1}\tendversN \dfrac{1}{u_2-1}.$$

(Oral Centrale PSI 2017) Correction

On définit la suite de réels $(x_n)$ par $x_0>0$ et, pour tout $n\in \N$, $x_{n+1}=x_n+\dfrac{1}{x_n}$. Donner la nature de $\dsum \dfrac{1}{x_n}$. Quel est le lien entre $x_{n+1}$ et $\dsum_{k=0}^n\dfrac{1}{x_k^2}$? Donner un équivalent de $x_n$.

Correction

Il est facile de voir que, pour tout $n\geq 0$, on a $x_n>0$ donc la suite est bien définie. D'autre part, on a $x_{n+1}-x_n=\dfrac{1}{x_n}>0$ donc la suite est strictement croissante.
On en déduit que la suite $(x_n)_n$ est soit convergente soit elle tend vers $\infty$, supposons que $x_n\tendversN\,\ell$, on aurait alors $\ell=\ell+\dfrac{1}{\ell}$ ce qui impossible. Conclusion $\boxed{x_n\tendversN\,\infty}$.
Ensuite, pour $n>0$, on a $$\dsum_{k=0}^n\frac{1}{x_k}=\dsum_{k=0}^n(x_{k+1}-x_k)=x_{n+1}-x_0\tendversN\,\infty\Longrightarrow\boxed{ \dsum\frac{1}{x_n}\text{ diverge}}.$$ %Soit $n\geq 1$, on a pour tout $k\in \N$, %$$x_{k+1}^2=x_k^2+2+\dfrac{1}{x_k^2}\Longrightarrow\dsum_{k=0}^nx_{k+1}^2-x_k^2=\dsum_{k=0}^n(2+\frac{1}{x_k^2})\Longrightarrow x_{n+1}^2-x_0^2=2(n+1)+\dsum_{k=0}^n\frac{1}{x_k^2}.$$ Soit $n\in \N$, on a $$x_{n+1}^2=x_n^2+2+\frac{1}{x_n^2}\Longrightarrow x_{n+1}^2-x_n^2=2+\frac{1}{x_n^2}\tendversN\,2 \text{ puisque }x_n\tendversN\infty.$$ On en déduit en utilisant le lemme d'escalier Conséquence de Lemme de Cesaro, $$x_n^2\underset{n\to\infty}{\sim}2n\Rightarrow \boxed{x_n\underset{n\to\infty}{\sim}\sqrt{2n}}.$$

(Centrale 2017) Correction

On donne $u_0=1,\,\dfrac{u_{n+1}}{u_n}=\dfrac{n+a}{n+b}$ où $a,\,b\in \R_+^*$, on pose $v_n=\ln\left(n^{b-a}u_n\right)$.
Montrer la convergence de la série $\dsum (v_{n+1}-v_n)$ et en déduire une condition sur $a$ et $b$ pour que $\dsum u_n$ converge.

Correction

On vérifie facilement que $u_n>0$ pour tout $n\in\N$, donc $v_n$ est bien définie.D'autre part, $$\begin{array}{lcl} v_{n+1}-v_n&=&\ln\left((n+1)^{b-a}u_{n+1}\right)-\ln\left(n^{b-a}u_n\right)=\ln\left(\left(1+\frac{1}{n}\right)^{b-a}\frac{u_{n+1}}{u_n}\right)\\ &=&\ln\left(\left[1+\frac{b-a}{n}+\frac{(b-a)(b-a-1)}{n^2}+\underset{n\to\infty}{\mathrm{o}}\left(\frac{1}{n^2}\right)\right]\left(1+\frac{a}{n}\right)\left[1-\frac{b}{n}+\frac{b^2}{n^2}+\underset{n\to\infty}{\mathrm{o}}\left(\frac{1}{n^2}\right)\right]\right)\\ &=&\ln\left(1+\dfrac{(b-a)(b+a-1)}{2n^2}++\underset{n\to\infty}{\mathrm{o}}\left(\frac{1}{n^2}\right)\right) = \dfrac{(b-a)(b+a-1)}{2n^2}++\underset{n\to\infty}{\mathrm{o}}\left(\frac{1}{n^2}\right) \end{array}$$ On en déduit que la série $\dsum v_{n+1}-v_n$ converge,ce qui implique que la suite $(v_n)$ tend vers une limite fini $\ell$. On en déduit alors, $$n^{b-a}u_n\tendversN\,\ee^\ell >0 \Longrightarrow u_n\underset{n\to\infty}{\sim}\dfrac{\ee^\ell}{n^{b-a}}$$ Conclusion: $$\boxed{ \dsum u_n \text{ converge ssi }b-a>1.}$$

(E3A?) Correction

On considère la suite $(u_n)_{n\in \N}$ définie par: $$u_0>1,\quad \forall n\geq 0,\,\,u_{n+1}=\dfrac{u_n^2+1}{2}.$$

  1. Montrer que $u_n\tendversN\,\infty$.
  2. On pose, pour $n\geq 0$, $v_n=\dfrac{\ln(u_n)-\ln(2)}{2^n}$ et $w_n=v_{n+1}-v_n$. Montrer que $\dsum w_n$ converge, en déduire qu'il existe $\ell \in \R$ tel que $v_n=\ell +\mathrm{o}\left(\frac{1}{2^n}\right)$.
  3. En déduire, qu'il existe $\lambda\in \R_+^*$ tel que $u_n\underset{n\to\infty}{\backsim}2\lambda^{2^n}$.

Correction

  1. Il est clair que la suite $(u_n)$ est strictement croissante, en effet, $$\forall n\in \N,\quad u_{n+1}-u_n=\dfrac{u_n^2+1}{2}-u_n=\dfrac{u_n^2-2u_n+1}{2}=\dfrac{(u_n-1)^2}{2}\geq 0,$$ or, puisque $\textcolor{blue}{u_0>1}$, donc $u_n\geq u_0>1$ pour tout $n\in \N$, ce qui donne $ u_{n+1}-u_n>0$.
    Supposons que $u_n\tendversN\,\ell$, on aurait alors $2\ell=\ell^2+1$,i.e. $\ell =1$ ce qui impossible, donc $(u_n)$ ne converge pas vers une limite finie, soit $\boxed{u_n\tendversN\,\infty}$.
  2. Soit $n\geq 0$, on a $$\begin{array}{lcl} w_n&=&v_{n+1}-v_n=\dfrac{\ln(u_{n+1})-\ln(2)}{2^{n+1}}-\dfrac{\ln(u_n)-\ln(2)}{2^n}\\ &=&\dfrac{\ln\left(\frac{u_n^2+1}{2}\right)-\ln(2)}{2^{n+1}}-\dfrac{\ln(u_n)-\ln(2)}{2^n}\\ &=& \dfrac{\ln\left(u_n^2+1\right)-2\ln(2)}{2^{n+1}}-\dfrac{2\ln(u_n)-2\ln(2)}{2^{n+1}}=\dfrac{\ln(u_n^2+1)-\ln(u_n^2)}{2^{n+1}}\\ &=&\dfrac{\ln\left(1+\frac{1}{u_n^2}\right)}{2^{n+1}} \end{array}$$ On en déduit, $2^nw_n=\dfrac{\ln\left(1+\frac{1}{u_n^2}\right)}{2}\tendversN\,0$, ce qui donne $w_n=\mathrm{o}\left(\frac{1}{2^n}\right)$ donc $\dsum w_n$ converge, notons $s$ sa somme.
    Soit $n>1$, on a $$\dsum_{k=0}^{n-1}w_k=\dsum_{k=0}^{n-1}v_{k+1}-v_k=v_{n}-v_0\tendversN\,s\Longrightarrow v_n\tendversN (s+v_0)=\ell.$$ D'autre part, $$v_n-\ell = v_n-v_0-s= \dsum_{k=0}^{n-1}w_k - s= \dsum_{k=0}^{n-1}w_k- \dsum_{k\geq 0}w_k= -\dsum_{k\geq n}w_k.$$
  3. Puisque $v_n\thicksim\ell$ et $\ln(u_n)-\ln(2)=2^nv_n$ on obtient: $$u_n=2\ee^{2^nv_n}\thicksim 2\ee^{2^n\ell}=2(\ee^\ell)^{2^n}.$$

(Oral Mines-Ponts P) Correction

Soit $(u_n)$ une suite à termes positifs telle que $\dsum u_n$ diverge. Donner la nature et la somme de $\dsum\dfrac{u_n}{(1+u_0)\cdots(1+u_n)}$.

Correction

Il suffit de remarquer que, pour $n\geq 1$, $$\dfrac{u_n}{(1+u_0)\cdots(1+u_n)}=\dfrac{1}{(1+u_0)\cdots(1+u_{n-1})}-\dfrac{1}{(1+u_0)\cdots(1+u_n)}. $$

Comaparaison série intégrale

(Cours) Correction

Soit $f$ une fonction définie sur $[1,\infty[$, positive et décroissante. On pose $$\forall n\in \N^*,\quad S_n=\dsum_{k=1}^nf(k),\quad I_n=\int_1^nf(t)\ud t,\text{ et } u_n=S_n-I_n.$$

  1. Montrer que la suite $(u_n)_n$ est décroissante et minorée, en déduire qu'elle est convergente.
  2. Montrer que sa limite $\ell \in [0,f(1)]$.
  3. Montrer que les limites des suites $$\mathbf{ a)}\, \quad 1+\dfrac{1}{2}+\cdots+\dfrac{1}{n}-\ln(n),\quad \mathbf{b)}\, \quad 1+\dfrac{1}{2^\alpha}+\cdots+\dfrac{1}{n^\alpha}-\int_1^n\dfrac{\ud t}{t^\alpha},\, (\alpha\in ]0,1[),$$ appartiennent à l'intervalle $]0,1[$.

Correction

Application directe du cours!

($\star\star$) Correction

Soit $\dsum_n a_n$ une série strictement positive et divergente. On suppose que $S_n = a_1+\cdots a_n > 1$ pour $n\in \N^*$. Vérifier les affirmations suivantes : $$\mathbf{a) }\, \dsum_{n\geq 1}\dfrac{a_{n+1}}{S_n\ln(S_n)} \text{ diverge},\quad\quad\quad \mathbf{b) }\, \dsum_{n\geq 1}\dfrac{a_{n}}{S_n\ln^2(S_n)} \text{ converge}.$$ On pourra remarquer que $a_{n+1}=S_{n+1}-S_n$

Correction

$\mathbf{a) }$ La fonction $\fonct{\varphi}{]1,\infty[}{\R_+}{x}{\frac{1}{x\ln(x)}}$ est strictement décroissante, en effet, $\forall x>1,\,\,\varphi'(x)=\dfrac{-\ln(x)-1}{(x\ln(x))^2}< 0$. De plus; $$\forall y > a >1,\,\dsp\int_a^y\varphi(t)\ud t=\left[\ln\left(\ln(t)\right)\right]_a^y=\ln(\ln(y))-\ln(\ln(a))\tendvers{y}{\infty}\infty.$$ On écrit pour $n\geq 1$, $$\dfrac{a_{n+1}}{S_n\ln(S_n)}=\dfrac{S_{n+1}-S_n}{S_n\ln(S_n)}\geq \int_{S_n}^{S_{n+1}}\dfrac{\ud x}{x\ln(x)}$$ On en déduit, pour tout $N>1$, $\dsp\dsum_{n=1}^N\dfrac{a_{n+1}}{S_n\ln(S_n)}\geq \int_{S_1}^{S_{N+1}}\dfrac{\ud x}{x\ln(x)}\tendvers{N}{\infty}\infty$ puisque $S_n\tendversN\infty$.
Donc $\boxed{\dsp \dsum_{n\geq 1}\dfrac{a_{n+1}}{S_n\ln(S_n)} \text{ diverge}}$.
$\mathbf{b) }$ La fonction $\fonct{\psi}{]1,\infty[}{\R_+}{x}{\frac{1}{\ln(x)}}$ est strictement décroissante, en effet, $\forall x>1,\,\,\psi'(x)=\dfrac{-1}{(x\ln(x))^2}< 0$. De plus; $$\forall y > a >1,\,\dsp\int_a^y\dfrac{1}{t\ln(t)^2}\ud t=\left[-\psi(t)\right]_a^y=\dfrac{1}{\ln(a)}-\dfrac{1}{\ln(y)}\tendvers{y}{\infty}\dfrac{1}{\ln(a)}.$$ Autrement dit, $\dsp \int_a^\infty \dfrac{1}{t\ln(t)^2}\ud t$ converge.
On écrit pour $n\geq 2$, $$\dfrac{a_{n}}{S_n\ln(S_n)^2}=\dfrac{S_{n}-S_{n-1}}{S_n\ln(S_n)^2}\leq \int_{S_{n-1}}^{S_{n}}\dfrac{\ud x}{x\ln(x)^2}\Longrightarrow 0\leq \dsum_{k=1}^n\dfrac{a_k}{S_k\ln(S_k)^2}\leq \dfrac{a_1}{S_1\ln(S_1)^2}+\int_{S_1}^\infty \dfrac{1}{t\ln(t)^2}\ud t.$$ Donc $\left(\dsum_{k=1}^n\dfrac{a_k}{S_k\ln(S_k)^2}\right)_n$ est majorée donc $\boxed{\dsp \dsum_{n\geq 1}\dfrac{a_{n}}{S_n\ln(S_n)^2} \text{ converge}}$.

(Cours) Correction

Soit $f\in \CC(\R_+,\R_+)$ décroissante, on suppose que $\dsp\int_{\R_+}f(x)\ud x$ existe.

  1. Justifier l'existence, pour tout $h>0$, de $\dsp\sum_{n=0}^\infty f(hn)$.
  2. Déterminer $\limiteX{h}{0^+}\dsp \sum_{n=0}^\infty f(hn)$.

Correction

$\mathbf{a) }$ Puisque $f$ est continue décroissante, on en déduit, pour tout $h>0$, $$\forall n\in \N,\forall x\in [n,n+1],~~f(h(n+1))\leq f(hx)\leq f(nh)\Longrightarrow f(h(n+1))\leq \int_n^{n+1}f(hx)\ud x\leq f(nh).$$ Notons, pour $n\in \N$, $S_n=\dsum_{k=0}^nf(hk)$, d'après la relation ci-dessus, on a: \begin{equation} %\label{Eq3D} \forall n>0,~~~\int_0^nf(hx)\ud x\leq S_n\leq \int_0^nf(hx)\ud x+f(0) \end{equation} Comme $\dsp\int_0^nf(hx)\ud x=\dfrac{1}{h}\int_0^{hn}f(u)\ud u < \dfrac{1}{h}\int_0^\infty f(x)\ud x< \infty$, on en déduit alors que $\dsum f(hn)$ converge.
$\mathbf{b) }$ Notons $S_h=\dsum_{n\geq 0} f(hn)$. On reprend la relation (\ref{Eq3D}), en faisant cet fois le changement de variable $t=hx$, on trouve: pour tout $ h>0$ et $ n >0$, \begin{equation} %\label{Eq4d} \int_0^{hn}f(t)\ud t\leq hS_n\leq \int_0^{hn}f(t)\ud t+hf(0)\Rightarrow \forall h>0,~\int_0^\infty f(t)\ud t\leq hS_h\leq \int_0^\infty f(t)\ud t +hf(0) \end{equation} On en déduit que $S_h\underset{h\,0^+}{\thicksim}\dfrac{A}{h}$ ou $A=\dsp\int_0^\infty f(t)\ud t$, donc $$\boxed{\,\limiteX{h}{0^+}\dsum_{n\geq 0}f(hn)=\infty\,}.$$

(Mines-Télécom PSI 2019) Correction

Soit $f$ une fonction continue, croissante et positive de $]0,1]$ dans $\mathbb{R}$. On note: $$\forall n\in \mathbb{N},\ \ u_n=f(\mathrm e^{-n}) \text{ et } \forall n\in\mathbb{N}^*,\ \ v_n=\dfrac{1}{n} f\left( \dfrac{1}{n} \right).$$ Montrer que les séries de terme général $u_n$ et $v_n$ ont même nature.

Correction

On définit les fonctions $\fonct{\varphi}{\R_+}{\R}{x}{f(\ee^{-x})}$ et $\fonct{\psi}{[1,\infty[}{\R}{x}{f(\frac{1}{x})/x}$.

Notons d'abord que les deux fonctions sont bien définies, puisque pour tout $x\geq 0$ on a $\ee^{-x}\in ]0,1]$ et pour tout $x\geq 1$, $\frac{1}{x}\in ]0,1]$.

Soient $x,y\in \R_+$, tels que $x\leq y$, on a $\ee^{-y}\leq \ee^{-x}$ puis $f(\ee^{-y})\leq f(\ee^{-x})$, puisque $f$ est croissante. Donc $\varphi$ est décroissante.

De même, soient $1\leq x\leq y$, on a $\frac{1}{y}\leq \frac{1}{x}$, donc $$\psi(y)=\dfrac{1}{y}f(\frac{1}{y})\leq \dfrac{1}{x}f(\frac{1}{y})\leq \dfrac{1}{x}f(\frac{1}{x})\leq \psi(x)$$ autremnt dit, $\psi$ aussi décroissante.

En utilisant les résultats du cours, on trouve: $$\dsum f(\ee^{-n}) \text{ et }\int_0^\infty f(\ee^{-t})\ud t \text{ sont de même nature}$$ et aussi $$\dsum \frac{1}{n}f(\frac{1}{n}) \text{ et }\int_1^\infty f(\frac{1}{t})\dfrac{\ud t}{t} \text{ sont de même nature}$$

Il faut maintenant lier les deux intégrales. Pour cela, on définit le changement de variable $u =\ln(t)$ avec $t\in [1,\infty[$, donc $u\in \R_+$, et $\ud u = \dfrac{ \ud t}{t}$, c'est un changement de variable admissible pour les intégrales impropres ci-dessous. En effet, la fonction $g(x)=\ln(x)$ est de classe $\CC^1$ strictement croissante et bijective).

On en déduit alors que les intégrales $$\int_0^\infty f(\ee^{-t}) \ud t \text{ et } \int_{g^{-1}(0)}^{g^{-1}(\infty)} f(\ee^{-g(x)})g'(x)\ud x \text{ sont de même nature}$$ Or le deuxième intégrale n'est rien d'autre que $\dsp\int_{1}^\infty \frac{f(1/x)}{x}\ud x$.

($\star\star$) Correction

Montrer que la série $\dsum \dfrac{n^2}{(1+n^2)^4}$ converge. Calculer une valeur approchée à $10^{-4}$ près de sa somme.

Correction

Pour tout $n\in \N$, on a $0\leq u_n=\dfrac{n^2}{(1+n^2)^4}\leq \dfrac{1}{n^6}$ et $\dsum\dfrac{1}{n^6}$ converge. Donc $\dsum u_n$ converge.
Pour trouver l'approximation de la somme il faut déterminer $N$ le nombre de termes à calculer. Notons $f(x)=\dfrac{x^2}{(1+x^2)^4}$, $f$ est continue positif strictement décroissante sur $[1,\infty[$ de plus $\dsp\int_1^\infty f(x)\ud x$ converge.
On a, $$\forall k\geq 2,~~f(k)=u_k\leq \int_{k-1}^k f(x)\ud x\Longrightarrow\sum_{k\geq n+1}u_k\leq \int_n^\infty f(x)\ud x\leq \int_n^\infty \dfrac{\ud x}{x^6}=\dfrac{1}{5n^5}$$ Il suffit alors de choisir $n$ tel que $\frac{1}{5n^5}\leq 10^{-4}$ i.e. $n=5$, d'où $$\dsum u_n=\dfrac{1}{2^4}+\dfrac{4}{3^4}+\dfrac{9}{10^4}+\dfrac{16}{17^4}+\dfrac{25}{26^4}+\varepsilon=0.070046+\varepsilon \text{ avec }\varepsilon\leq 10^{-4}.$$

($\star\star$) Correction

Soit $(a_n)$ une suite réelle, on suppose que $(a_n)$ est strictement décroissante et $a_n\tendversN\ell >0$. Montrer que $\dsum\dfrac{a_n-a_{n+1}}{a_n}$ converge.

Correction

Vu les hypothèse sur $(a_n)$, on en déduit que, pour tout $n\in \N$, $a_n\geq \ell >0$, donc $\dfrac{a_n-a_{n+1}}{a_n}$ est bien définie.
Soit $n\in \N$, on a pour tout $t\in ]a_{n+1},a_n[$, $\dfrac{1}{a_n}\leq \dfrac{1}{t}$, on en déduit, $$\forall n\geq 0,\,0\leq \dfrac{a_n-a_{n+1}}{a_n}\leq \int_{a_{n+1}}^{a_n}\dfrac{\ud t}{t}=\ln\left(\dfrac{a_n}{a_{n+1}}\right).$$ En additionnant la relation précédente, pour $n\in \inter{0,N}$ ($N\geq 1$), on trouve $$0\leq \dsum_{n=0}^N\dfrac{a_n-a_{n+1}}{a_n}\leq \dsum_{n=0}^N\int_{a_{n+1}}^{a_n}\dfrac{\ud t}{t}=\ln\left(\dfrac{a_0}{a_{N}}\right)< C.$$ En effet, puisque $a_n\tendversN\ell >0$, donc $\ln\left(\dfrac{a_0}{a_{N}}\right)$ tend vers une limite finie strictement positive, donc cette suite est majorée par une constante $C>0$.
On en déduit alors $\dsum\dfrac{a_n-a_{n+1}}{a_n}$ converge

($\star\star$) Correction

On considère la série $\dsum \dfrac{1}{n^3}$ (une série convergente), on note $S$ sa somme, et pour tout $n\geq 1$, on note $$S_n=\dsum_{k=1}^n\dfrac{1}{k^3}\quad \text{et}\quad R_n=\dsum_{k\geq n+1}\dfrac{1}{k^3}.$$

  1. A l'aide d'une comparaison avec une intégrale, montrer que: $$\forall n\geq 1,\quad \dsp\dfrac{1}{2(n+1)^2}\leq R_n\leq \dfrac{1}{2n^2}.$$ En déduire le plus petit entier $n$ tel que $\abs{S-S_n}\leq 10^{-6}$.
  2. On pose, pour $n\geq 1$, $A_n=S_n+\dfrac{1}{2}\left(\dfrac{1}{2(n+1)^2}+\dfrac{1}{2n^2}\right)$. En utilisant la question précédente, montrer que: $$\forall n\in \N^*,\quad \abs{A_n-S}\leq \dfrac{1}{2n^3}.$$ En déduire le plus petit entier $n$ tel que $\abs{S-A_n}\leq 10^{-6}$.

Correction

  1. La fonction $f:x\longmapsto \dfrac{1}{x^3}$ est strictement décroissante sur $\R_+^*$. $$\forall k\geq 2,\, \int_k^{k+1}f(x)\ud x\leq \dfrac{1}{k^3}\leq \int_{k-1}^kf(x)\ud x$$ Soient $ N>n\geq 1$, on a: $$ \int_{n+1}^{N+1}f(x)\ud x\leq \dsum_{k=n+1}^N\dfrac{1}{k^3}\leq \int_n^N f(x)\ud x$$ Ce qui donne $$ \dfrac{1}{2(n+1)^2}-\dfrac{1}{2(N+1)^2}\leq \dsum_{k=n+1}^N\dfrac{1}{k^3}\leq \dfrac{1}{2n^2}-\dfrac{1}{2N^2}.$$ En faisant $N$ tend vers $\infty$, on trouve $$\forall n\geq 1,\quad \dsp\dfrac{1}{2(n+1)^2}\leq R_n\leq \dfrac{1}{2n^2}.$$ Ainsi, si on souhaite que $R_n$ soit inférieur à $10^{-6}$ il suffit de prendre $n$ tel que $\dfrac{1}{2n^2}\leq 10^{-6}$ soit $$\textcolor{red}{n\geq 708}.$$
  2. Soit $n\geq 1$, on a $$\begin{array}{lcl} \abs{S-A_n}&=&\abs{S_n+R_{n}-\left(S_n+\dfrac{1}{2}\left(\dfrac{1}{2(n+1)^2}+\dfrac{1}{2n^2}\right)\right)}\\ &&\\ &=&\abs{R_n-\dfrac{1}{2}\left(\dfrac{1}{2(n+1)^2}+\dfrac{1}{2n^2}\right)}\\ &&\\ &\leq &\dfrac{1}{2}\left(+\dfrac{1}{2n^2}-\dfrac{1}{2(n+1)^2}\right)=\dfrac{1}{4}\dfrac{2n+1}{n^2(n+1)^2}\leq \dfrac{1}{2n^3} \end{array}$$ Ainsi, si on souhaite que $\abs{S-A_n}$ soit inférieur à $10^{-6}$ il suffit de prendre $n$ tel que $\dfrac{1}{2n^3}\leq 10^{-6}$ soit $$\textcolor{blue}{n\geq 80}.$$

Conclusion, pour avoir une valeur approchée de $S$ à $10^{-6}$ près, il suffit de calculer $S_{80}$ puis ajouter $\dfrac{1}{4}\left(\dfrac{1}{6400}+\dfrac{1}{6561}\right)$.
En utilisant le logiciel Maple, on trouve les résultats suivants: $$\begin{array}{lcl} S &=&1.202\,\,056\,\,903\,\,159\,\,594\,\,285\,\,399\,\,\cdots\\ S_{708}&=&1.202\,\,055\,\,907\,\,089\,\,092\,\,414\,\,947\,\,\cdots \\ S_{80} &=&1.201\,\,979\,\,748\,\,618\,\,896\,\,502\,\,181\,\,\cdots\\ A_{80} &=&1.202\,\,056\,\,915\,\,066\,\,465\,\,470\,\,327\,\,\cdots\\ R_{708}&=& .9960705018704517858608418878894904215906704e-6\\ \abs{S-A_{80}}&=&.119068711849273283819093965990942375626486e-7 \end{array}$$

(Centrale 2016) Correction

Soit $\alpha \in \R$, pour tout $n\in \N^*$, on pose $u_n=\dsum_{k=1}^n\dfrac{1}{(n+k)^\alpha}$.

  1. Déterminer la limite de la suite $(u_n)_n$ selon la valeur de $\alpha$.
  2. On se place dans le cas ou $\alpha>1$.
    1. Montrer que $u_n\tendversN \,0$.
    2. Déterminer la nature de la série de terme générale $v_n$ avec $v_n=\dsum_{k\geq 1}\dfrac{1}{(n+k)^\alpha}$.

Correction

  1. Il est clair que $u_n\tendversN\,\infty$ pour $\alpha\leq 0$.
    Supposons que $\alpha>0$, alors $$\forall k\in \inter{1,n},\quad \dfrac{1}{(2n)^\alpha}\leq \dfrac{1}{(n+k)^\alpha}\leq \dfrac{1}{(n+1)^\alpha} \Longrightarrow \dfrac{n}{2^\alpha n^\alpha}\leq u_n\leq \dfrac{n}{(n+1)^\alpha}\leq n^{1-\alpha}.$$ Ainsi, $$\alpha\in ]0,1[,\quad u_n\tendversN\,\infty,\,\alpha>1,\quad u_n\tendversN\,0.$$ Pour $\alpha=1$, on a $$u_n =\dfrac{1}{n}\dsum_{k=1}^n\dfrac{1}{1+k/n}\tendversN\,\int_0^1\dfrac{\ud x}{1+x}=\ln(2).$$
  2. Pour $\alpha>1$, on a déjà montré que $u_n\tendversN\,0$. En utilisant la comparaison série-intégrale, on a $$\forall n>1,\, v_n=\dsum_{k\geq 1}\dfrac{1}{(n+k)^\alpha}=\dsum_{k\geq n+1}\dfrac{1}{k^\alpha}\underset{n\to \infty}{\thicksim}\dfrac{1}{\alpha-1}\dfrac{1}{n^{\alpha-1}}.$$ Ainsi, $\dsum v_n$ converge ssi $\alpha>2$.

($\star\star\star$) Correction

Soit $f\in \CC^1(\R_+^*,\R_+^*)$, croissante et bijective. On note $g=f^{-1}$. Montrer que: $$\dsum\frac{1}{f(n)}\text{ converge } \Longleftrightarrow \dsum\frac{g(n)}{n^2} \text{ converge}.$$

Correction

Soient $0< a < b$, on a $$\int_a^b\dfrac{\ud t}{f(t)}=\left[\dfrac{t}{f(t)}\right]_a^b +\int_a^b\dfrac{tf'(t)\ud t}{f^2(t)},\text{ avec }\left\{\begin{array}{lcl} u'(t)=1&&u(t)=t\\ v(t)=\dfrac{1}{f(t)}&&v'(t)=\dfrac{-f'(t)}{f(t)^2} \end{array} \right. $$ Puis dans le deuxième intégrale, on pose $u=f(t)$ (i.e. $t=g(u)$), ce qui donne $\ud u =f'(t)\ud t$, on obtient alors $$\int_a^b\dfrac{\ud t}{f(t)}=\dfrac{b}{f(b)}-\dfrac{a}{f(a)}+\int_{f(a)}^{f(b)}\dfrac{g(u)\ud u}{u^2}. \quad\quad\quad (\star)$$ $\mathbf{1 . }$ Supposons que $\dsum\dfrac{1}{f(n)}$ converge
D'après le cours $\dsp\int_a^\infty\dfrac{\ud t}{f(t)}$ converge, donc $\dsp\limiteX{b}{\infty}\dsp\int_a^b\dfrac{\ud t}{f(t)}$ existe. Ainsi $$\limiteX{b}{\infty}\int_{f(a)}^{f(b)}\dfrac{g(u)\ud u}{u^2}\text{ et }\limiteX{b}{\infty}\dfrac{b}{f(b)}\text{ sont de même nature}.$$ Or puisque $\dsum \dfrac{1}{f(k)}$ converge alors $\dsum_{k=n}^{2n}\dfrac{1}{f(k)}\tendversN\,0$, ce qui donne $$\forall n\in \N,\quad\dfrac{2n}{f(2n)}= \dfrac{1}{2}\dsum_{k=n}^{2n}\dfrac{1}{f(2n)}\leq \dfrac{1}{2}\dsum_{k=n}^{2n}\dfrac{1}{f(k)}\tendversN\,0$$ De même, on montre que $\dfrac{2n+1}{f(2n+1)}\tendversN\,0$, ce qui implique que $\dfrac{n}{f(n)}\tendversN\,0$.
Soit $b>0$, on note $n=E(b)$, alors $$\dfrac{n-1}{f(n+1)}\leq \dfrac{b}{f(b)}\leq \dfrac{n+1}{f(n)} $$ donc $\dsp\lim_{b\to \infty}\dfrac{b}{f(b)}=0$. Donc $\dsp \int_{f(a)}^\infty\dfrac{g(u)\ud u}{u^2}$ converge.
Mais ceci ne nous permet pas de conclure car la fonction $t\longmapsto\dfrac{g(t)}{t^2}$ à priori n'est décroissante, néanmoins on peut écrire (puisque $g$ est croissante), $$\dfrac{g(n)}{(n+1)^2}\leq \int_n^{n+1}\dfrac{g(t)\ud t}{t^2} \Longrightarrow \dsum_{n}\dfrac{g(n)}{(n+1)^2} < \infty.$$ On en déduit alors que $\dsum \dfrac{g(n)}{n^2}$ converge.
$\mathbf{2 .}$ Inversement, supposons que $\dsum \dfrac{g(n)}{n^2}$ converge, alors $\dsp\int_1^\infty\dfrac{g(t)\ud t}{t^2}$ converge, en effet, on a $$\forall n>0,\quad 0\leq \int_n^{n+1}\dfrac{g(t)\ud t}{t^2}\leq \dfrac{g(n+1)}{n^2}.$$ En utilisant la relation $(\star)$, on trouve $$\forall b>1,\quad \int_1^b\dfrac{\ud t}{f(t)}-\dfrac{b}{f(b)}=-\dfrac{1}{f(1)}+\int_{f(1)}^{f(b)}\dfrac{g(t)\ud t}{t^2}$$ En particulier $\dsp \int_1^b\dfrac{\ud t}{f(t)}-\dfrac{b}{f(b)}$ admet une limite finie lorsque $b$ tend vers $\infty$, donc $\dsp \int_1^b\dfrac{\ud t}{f(t)}$ et $\dfrac{b}{f(b)}$ sont de même natures.
La fonction $b\mapsto \dsp \int_1^b\dfrac{\ud t}{f(t)}$ est postives, croissante, donc soit elle admet une limite finie lorsque $b$ tend vers $\infty$ soit elle tend vers $\infty$.
Supposons que $\dsp \int_1^b\dfrac{\ud t}{f(t)}\tendvers{b}{\infty}\infty$, alors $\dfrac{b}{f(b)}\tendvers{b}{\infty}\infty$. Alors, $$ \exists n_0,\quad \forall n\geq n_0,\quad n\geq f(n)$$ La fonction $g$ est croissante, donc pour tout $n\geq n_0$, on a $$ g(n)\geq g(f(n))=n\Longrightarrow \forall n\geq n_0,\quad \dfrac{g(n)}{n^2}\geq \dfrac{1}{n}$$ Ce qui est impossible car $\dsum g(n)/n^2$ converge.
On en déduit alors que $\dsp \int_1^b\dfrac{\ud t}{f(t)}$ converge vers une limite finie, donc d'après le cours $\dsum \dfrac{1}{f(n)}$ converge.

(X-ENS Cachan (PSI) 2019) Correction

Soit $\left( u_{n}\right) \in \left( \mathbb{R}^*_+\right) ^{\mathbb{N}}$, de limite nulle et $\alpha >1$. On suppose que $\lim\limits_{n\rightarrow\infty }\dfrac{u_{n}-u_{n+1}}{u_{n}^{\alpha }}=\ell \in \mathbb{R}^{\ast }$. L'objectif est de montrer que la série de terme général (s.t.g.) $u_{n}$ converge si et seulement si $\alpha < 2$.

  1. Montrer que $\left( u_{n}\right)$ est strictement décroissante à partir d'un certain rang $N$.
  2. Soit $\alpha <2$. À l'aide de l'inégalité suivante (que l'on justifiera) : $\dfrac{u_{n}-u_{n+1}}{u_{n}^{\alpha -1 }}\leqslant \displaystyle{\int_{u_{n+1}}^{u_{n}}}\dfrac{\ud t}{t^{\alpha -1}}$.
    Montrer que la s.t.g. $\textcolor{red}{\dfrac{u_{n}-u_{n+1}}{u_{n}^{\alpha -1}}}$ converge. En déduire la convergence de la s.t.g. $u_{n}$.
  3. Soit $\alpha \geqslant 2$. Montrer que la s.t.g. $\dfrac{u_{n}-u_{n+1}}{u_{n}^{\alpha -1}}$ diverge et en déduire que la s.t.g. $u_{n}$ diverge également.
Correction

  1. Puisque $\ell \neq 0$, alors il y deux cas: soit $\ell < 0$ soit $\ell >0$.
    Supposons que $\ell < 0$, alors il existe $n_0$ tel que pour tout $n\geq n_0$ on a $\dfrac{u_n-u_{n+1}}{u_n^\alpha} < 0$, donc (puisque $u_n^\alpha >0$) $u_n-u_{n+1}< 0$, autrement dit la suite $(u_n)$ est strictement croissante (à partir d'un certain rang), en particulier, $$\forall n\geq n_0,\quad u_n> u_{n_0}>0 \Longrightarrow \lim_{n\to \infty} u_n >0$$ ce qui contredit l'hypothèse de l'enoncé.
    On en déduit alors que $\ell >0$, donc il existe $n_0$ tel que pour tout $n\geq n_0$ on a $\dfrac{u_n-u_{n+1}}{u_n^\alpha} > 0$, donc $u_n-u_{n+1}> 0$, autrement dit la suite $(u_n)_{n\geq n_0}$ est strictement décroissante.
  2. Soit $\alpha\in ]1,2[$, la fonction $t\longmapsto \dfrac{1}{t^{\alpha-1}}$ est strictement décroissante sur $\R_+^*$. Soit $n\geq n_0$, on a $$\forall t\in [u_{n+1},u_n],\, \dfrac{1}{u_n^{\alpha-1}}\leq \dfrac{1}{t^{\alpha-1}}\Longrightarrow \int_{u_{n+1}}^{u_n}\dfrac{\ud t}{u_n^{\alpha-1}}\leq \int_{u_{n+1}}^{u_n} \dfrac{\ud t}{t^{\alpha-1}}.$$ Ce qui donne, $$\forall n\geq n_0,\quad 0< \dfrac{u_n-u_{n+1}}{u_n^{\alpha-1}}\leq \int_{u_{n+1}}^{u_n}\dfrac{\ud t}{t^{\alpha-1}} =\dfrac{1}{2-\alpha} \left(u_n^{2-\alpha}-u_{n+1}^{2-\alpha}\right)$$ Comme $2-\alpha>0$ alors $u_n^{2-\alpha}\tendversN\,0$, donc la série téléscopique $\dsum_{n\geq n_0} u_n^{2-\alpha}-u_{n+1}^{2-\alpha}$ converge, ce qui implique donc que $\dsum \dfrac{u_n-u_{n+1}}{u_n^{\alpha-1}}$ converge.
    D'autre part, on a $$\ell \underset{n\to \infty}{\thicksim} \dfrac{u_n-u_{n+1}}{u_n^{\alpha}}\Longrightarrow \ell u_n {\thicksim} \dfrac{u_n(u_n-u_{n+1})}{u_n^{\alpha}} =\dfrac{u_n-u_{n+1}}{u_n^{\alpha-1}}$$ donc $\dsum u_n$ et $\dsum \dfrac{u_n-u_{n+1}}{u_n^{\alpha-1}}$ sont de même nature.
  3. On suppose que $\alpha > 2$, en utilisant la même démarche que dans la question précédente, on trouve: $$\forall n\geq n_0,\quad \int_{u_{n+2}}^{u_{n+1}}\dfrac{\ud t}{t^{\alpha-1}}\leq \int_{u_{n+1}}^{u_n}\dfrac{\ud t}{u_n^{\alpha-1}}$$ ce qui donne $$ \dfrac{u_n-u_{n+1}}{u_n^{\alpha-1}}\geq \dfrac{1}{2-\alpha} \left(u_{n+1}^{2-\alpha}-u_{n+2}^{2-\alpha}\right) \quad \quad (\star)$$ La série téléscopique $ \dsum u_{n+1}^{2-\alpha}-u_{n+2}^{2-\alpha}$ diverge puisque $u_{n+1}^{2-\alpha}\tendversN\infty$, on en déduit alors que $\dsum \dfrac{u_n-u_{n+1}}{u_n^{\alpha-1}}$ diverge aussi.
    Le cas $\alpha=2$ est similaire, il faut juste remplacer (dans $\star$) le terme $\dfrac{1}{2-\alpha} \left(u_{n+1}^{2-\alpha}-u_{n+2}^{2-\alpha}\right)$ par $\ln(u_{n+1})-\ln(u_{n+2})$.

Produit de Cauchy

($\star$) Correction

Montrer la convergence de la série de terme général $w_n=\dsum_{p=1}^n\dfrac{1}{p^2(n-p)!}$ et donner sa somme.

Correction

Pour $n\in \N$, on pose $$u_0=0,\,\, u_n=\dfrac{1}{n^2} \quad n\geq 1,\quad \text{ et } v_n=\dfrac{1}{n!}$$ Les deux séries $\dsum u_n$ et $\dsum v_n$ sont absolument convergentes, donc la série produit de Cauchy de ces deux séries est absolument convergente. Notons $w_n$ le terme générale de la série produit, alors $$\forall n\in \N,\quad w_n=\dsum_{k=0}^n u_kv_{n-k}=\dsum_{k=1}^n \dfrac{1}{k^2}\dfrac{1}{(n-k)!}.$$ En utilisant le résultat de cours, on trouve: $$\dsum_{n\geq 1}w_n=\left(\dsum_{n\geq 0}u_n\right)\left(\dsum_{n\geq 0}v_n\right)=\dfrac{\pi^2}{6}\ee.$$

($\star\star$) Correction

Étudier la convergence du produit de Cauchy de la série $\dsum_{n\geq 1}\dfrac{(-1)^{n-1}}{\sqrt{n}}$ par elle-même.

Correction

Pour $n\geq 1$ on note $u_n=\dfrac{(-1)^n}{\sqrt{n}}$ et $u_0=0$, alors le terme générale $w_n$ de la série produit de Cauchy de la série $\dsum_{n\geq 1}\dfrac{(-1)^{n-1}}{\sqrt{n}}$ par elle-même, est $$\forall n\geq 1,\quad w_n=\dsum_{k=0}^{n}u_ku_{n-k}=\dsum_{k=1}^{n-1}\dfrac{(-1)^{k-1}(-1)^{n-k-1}}{\sqrt{k}\sqrt{n-k}}=\dfrac{(-1)^n}{n}\dsum_{k=1}^{n-1}\dfrac{1}{\sqrt{(k/n)(1-(k/n))}}$$ or pour $k\in \inter{1,n-1}$, $\frac{k}{n}\in ]0,1[$ donc $$(k/n)(1-(k/n))\leq \dfrac{1}{4}\Longrightarrow\dfrac{1}{\sqrt{(k/n)(1-(k/n))}}\geq 2$$ (il suffit d'étudier $x\longmapsto x(1-x)$ sur l'intervalle $]0,1[$.
On en déduit alors, $$\forall n\in \N^*,\quad \abs{w_n}=\dfrac{1}{n}\dsum_{k=1}^{n-1}\dfrac{1}{\sqrt{(k/n)(1-(k/n))}}\geq \dfrac{1}{n}\dsum_{k=1}^{n-1}2=\frac{2n-2}{n}$$ Ainsi $w_n$ ne tend pas vers $0$ lorsque $n$ tend vers $\infty$, conclusion $\dsum w_n$ diverge.

($\star$) Correction

Étudier la convergence du produit de Cauchy de la série $\dsum_{n\geq 1}\dfrac{(-1)^n}{n}$ par elle-même.

Correction

Pour $n\geq 1$ on note $u_n=\dfrac{(-1)^n}{n}$ et $u_0=0$, alors le terme générale $w_n$ de la série produit de Cauchy de la série $\dsum_{n\geq 1}\dfrac{(-1)^{n}}{n}$ par elle-même, est $$\begin{array}{lcl} \forall n\geq 1,\quad w_n&=&\dsum_{k=0}^{n}u_ku_{n-k}=\dsum_{k=1}^{k}\dfrac{(-1)^{k}(-1)^{n-k}}{k(n-k)}\\ &=&(-1)^n\dsum_{k=1}^{n-1}\dfrac{1}{k(n-k)}=(-1)^n\dsum_{k=1}^{n-1}\left(\dfrac{1/n}{k}+\dfrac{1/n}{(n-k)}\right)\\ &=&\dfrac{(-1)^n}{n}\left(\dsum_{k=1}^{n-1}\dfrac{1}{k}+\dsum_{k=1}^{n-1}\dfrac{1}{(n-k)}\right)=\dfrac{2(-1)^n}{n}\dsum_{k=1}^{n-1}\dfrac{1}{k}\\ \end{array}$$ La suite $(\abs{w_n})$ est décroissante et tend vers $0$ (car $\dsum_{1}^n\frac{1}{k}\backsim \ln(n)$), donc d'après le critère spécial des séries alternées, $\dsum w_n$ converge.

($\star$) Correction

Soit $u_n = \dsp \dfrac{(-1)^n}{(n+1)^2}$ et $v_n= \dsp \dfrac{(-1)^n}{n+1}$. Soit $w_n$ le terme général de la série produit de Cauchy des deux séries de $\dsum u_n$ et $\dsum v_n$. Étudier la convergence et la convergence absolue de $\dsum w_n$.

Correction

$w_n=(-1)^n \dsum_{k=0}^n\dfrac{1}{(k+1)^2(n-k+1)}$. En décomposant la fraction rationnelle (de $k$) en éléments simples, on trouve: $$\begin{array}{lcl} \dfrac{1}{(k+1)^2(n-k+1)}&=&\dfrac{\alpha k +\beta }{(k+1)^2}+\dfrac{\gamma}{(n-k+1)}\\ &=&\dfrac{(\gamma-\alpha)k^2+((n+1)\alpha+2\gamma-\beta)k+\gamma+(n+1)\beta}{(k+1)^2(n-k+1)} \end{array}$$ Donc, par identification, on a $$\dfrac{1}{(k+1)^2(n-k+1)}=\dfrac{-1/(n+2)^2 k +(n+3)/(n+2)^2 }{(k+1)^2}+\dfrac{1/(n+2)^2}{(n-k+1)}$$ ce qui donne, $$\begin{array}{lcl} w_n&=&\dfrac{(-1)^n}{n+2}\dsum_{k=0}^n \dfrac1{(k+1)^2}+\dfrac{(-1)^n}{(n+2)^2}\dsum_{k=0}^n \left(\dfrac1{k+1}+\dfrac1{n-k+1}\right)\\ &&\\ &=& \dfrac{(-1)^n}{n+2}\dsum_{k=0}^n \dfrac1{(k+1)^2}+\dfrac{2(-1)^n}{(n+2)^2}\dsum_{k=1}^{n+1}\dfrac1k \end{array} $$ Le premier terme de cette somme est équivalent à $(-1)^n\dfrac{\pi^2}{6}\dfrac1{n}$ et le second à $2(-1)^n\dfrac{\ln n}{n^2}$.
On en déduit $\dsum w_n$ est convergente, mais pas absolument convergente.

($\star$) Correction

Soient $(a,b)\in \C^2$ tel que $\abs{a}< 1$ et $\abs{b}< 1$. Montrer que : $$a\neq b\Longrightarrow \dfrac{1}{(1-a)(1-b)}=\dsum_{k\geq 0}\dfrac{a^{k+1}-b^{k+1}}{a-b},\text{ et } \dfrac{1}{(1-a)^2}=\dsum_{k\geq 0} (k+1)a^{k}.$$

Correction

Soient $(a,b)\in \C^2$ tel que $\abs{a}< 1$ et $\abs{b}< 1$ et $a\neq b$, on suppose de plus que $b\neq 0$ (en effet si $b=0$ il n'y a rien à démontrer).
D'après le cours, on a $\dfrac{1}{1-a}=\dsum_{n\geq 0}a^n$ et $\dfrac{1}{1-b}=\dsum_{n\geq 0}b^n$ avec convergence absolu pour les deux séries. Donc la série $\dsum w_n$ produit de Cauchy de ces deux séries converge, avec $$\begin{array}{lcl} \forall n\in \N,\,w_n&=&\dsum_{k=0}^na^kb^{n-k}\\ &=&b^n\dsum_{k=0}^n(\frac{a}{b})^k=b^n\dfrac{1-(\frac{a}{b})^{n+1}}{1-\frac{a}{b}}=\dfrac{b^n-\frac{a^{n+1}}{b}}{1-\frac{a}{b}}\\ &=&\dfrac{b^{n+1}-a^{n+1}}{b-a} \end{array}$$ On en déduit,d'après le cours, $$\boxed{\dfrac{1}{(1-a)(1-b)}=\left(\dsum_{n\geq 0}a^n\right)\left(\dsum_{n\geq 0}b^n\right)=\dsum_{k\geq 0}\dfrac{a^{k+1}-b^{k+1}}{a-b}}.$$ Dans le cas ou $a=b$, on aura $$\forall n\in \N,\,w_n=\dsum_{k=0}^na^ka^{n-k}=a^n\dsum_{k=0}^n1=(n+1)a^n,$$ Ce qui donne $\boxed{\dsp \dfrac{1}{(1-a)^2}=\dsum_{k\geq 0} (k+1)a^{k}.}$

($\star$) Correction

Soit $a\in ]-1,1[$. Montrer que la série $\dsum (n+1)a^n$ converge et calculer sa somme.

Correction

On considère la série $U=\dsum a^n$, on sait que c'est une série absolument convergente (puisque $\abs{a}< 1$). On note $W$ la série produit de Cauchy de $U$ par elle même. $$\forall n\geq 0,\quad w_n=\dsum_{k=0}^n u_ku_{n-k}=\dsum_{k=0}^n a^ka^{n-k}=\dsum_{k=0}^n a^n=(n+1)a^n.$$ En utilisant le résultat du cours, on a $W$ est absolument convergente et $W=U^2$, c-à-d $$\forall a\in ]-1,1[,\quad \dsum_{n=0}^\infty (n+1)a^n=\left(\dsum_{n=0}^\infty a^n\right)^2=\dfrac{1}{(1-a)^2}.$$